Контрольная по физике расчет количества теплоты: Контрольная работа №1 по теме «Расчет количества теплоты» ( 8 класс)

Содержание

Контрольная работа №1 по теме «Расчет количества теплоты» ( 8 класс)

Урок 11. Контрольная работа №1 по теме «Расчет количества теплоты»

Цель: проверить степень усвоения знаний, умений и навыков учащихся по теме «Расчёт количества теплоты»

Задачи:

1.  Проверить уровень усвоения понятий: нагревание, охлаждение, удельная теплоёмкость вещества, удельная теплота сгорания топлива.

2.  Определить уровень сформированности умений учащихся анализировать физическую ситуацию, записывать данные используя обозначения физических величин, выполнять перевод единиц, применять формулы для вычислений искомых физических величин.

3.  Оценить навыки учащихся при работе с таблицами «Удельная теплоёмкость вещества», «Удельная теплота сгорания топлива»

Ход урока.

1. Организационный момент. Приветствие. Выявление отсутствующих. Проверка готовности к контрольной работе (наличие тетради, ручки, калькулятора).

2. Выполнение контрольной работы.

Раздается учащимся контрольная работа из КИМ (4 варианта). В тетрадях записывают дату, тему контрольной работы.

А.Е.Марон, Е.А. Марон Контрольные и самостоятельные работы по физике 8 класс: к учебнику А.В. Перышкина. Физика

Критерии оценивания:

Контрольная работа состоит из 4 вариантов. Каждый вариант содержит блоки задач разных уровней сложности, которые отделены друг от друга чертой. Первый и второй уровни сложности (I и II) соответствуют требованиям обязательного минимума содержания физического образования в основной школе, третий (III) уровень предусматривает углублённое изучение физики. Поэтому, для того чтобы получить оценку «отлично» учащимся предлагается выполнить все задачи из I уровня и две (выбрать любые) из II уровня сложности.

Задания из III уровня сложности являются не обязательными, но могут выбираться учащимися по желанию.

Работа оценивается отметкой «4». если выполнен правильно весь I уровень и одна задача из блока задач второго уровня сложности. 7)

_____________________________________________________________________________

4. Для нагревания 400 г свинца от 25 до 45 ºС требуется количество теплоты 1120 Дж. Определите удельную теплоемкость свинца. ( 140Дж/кг*С)

5. Какое количество теплоты потребуется для того, чтобы в алюминиевом чайнике массой 700 г вскипятить 2 кг воды? Начальная температура воды 20 ºС. ( 723520 Дж).

6. На сколько градусов нагреется 4 кг воды при сжигании 30 г каменного угля, если считать, что вся энергия, выделенная при сгорании угля, пойдет на нагревание воды?

( 48, 2 С)

_____________________________________________________________________________

Контрольная работа №1 по теме «Расчет количества теплоты»

Вариант 2

1. Какое количество теплоты требуется для нагревания кирпича массой 4 кг от 15 до 30 ºС? ( 52 800 Дж)

2. Какое количество теплоты отдал окружающей среде кипяток массой 3 кг при остывании до 50 ºС? ( 630 000 Дж)

3. Сколько энергии выделится при полном сгорании 4 т каменного угля? ( 108Гдж)

____________________________________________________________________________

4. Воду какой массы можно нагреть от 0 до 60 ºС, сообщив ей количество теплоты 500 кДж? ( 1,98 кг)

5. Определите, какое количество теплоты потребуется для нагревания смеси из 300 г воды и 50 г спирта от 20 до 70 ºС. ( 69250 Дж)

6. Сколько граммов спирта потребуется, чтобы нагреть до кипения 3 кг воды, взятой при температуре 20 ºС? Потерями тепла пренебречь. ( 37,3 г)

_____________________________________________________________________________

Контрольная работа №1 по теме «Расчет количества теплоты»

Вариант 3

1. При обработке алюминиевой детали на станке температура её повысилась от 20 до 420 ºС. На сколько при этом изменилась внутренняя энергия детали, если её масса 500 г? ( 184000 Дж).

2. Какое количество теплоты выделится при охлаждении на 80 ºС свинцовой детали массой 400 г? ( 4 480 Дж). 7)

_____________________________________________________________________________

4. На сколько градусов охладится 40 г льда, если он при этом отдает количество теплоты 500 Дж? ( 5,95 С)

5. Алюминиевая кастрюля массой 250 г вмещает 2 кг молока. Какое количество теплоты требуется для нагревания в этой кастрюле молока от 15 до 100 ºС? Удельная теплоемкость молока = 3900 Дж\кг*С. ( 682550 Дж).

6. Рассчитайте массу керосина, который потребуется сжечь для того, чтобы нагреть 10 кг воды от 10 до 80 ºС, если вся энергия, выделенная при сгорании керосина, пойдет на нагрев воды. ( 0,64 кг)

_____________________________________________________________________________

Контрольная работа №1 по теме «Расчет количества теплоты»

Вариант 4

1. Какое количество теплоты выделяется при остывании 3 т чугуна на 100 ºС? ( 162 МДж)

2. Какое количество теплоты необходимо для нагревания от 10 до 40 ºС латунной гири массой100 г?

3. Определите количество теплоты, выделяющееся при сжигании 50 кг дров.

_____________________________________________________________________________

4. Сколько граммов стали можно нагреть на 20 ºС, сообщив ей количество теплоты 1500 Дж?

5. В железный душевой бак, масса которого 60 кг, налили холодной воды массой 100 кг. Под действием солнечного излучения температура воды повысилась от 4 до 30 ºС. Какое количество теплоты получили баки вода?

6. Воду какой массы можно нагреть от 30 ºС до кипения, израсходовав 2 кг дров? Потерями тепла пренебречь.

Контрольная работа по физике «Расчет количества теплоты при теплообмене» (8 класс)

Контрольная работа

по теме «Расчет количества теплоты при теплообмене»

Вариант______________

  1. Для нагревания кирпича массой 4 кг от 150С до 300С израсходовано 48 кДж теплоты. Найти удельную теплоемкость кирпича.

  1. Какое количество теплоты потребуется для нагревания 2 кг воды в алюминиевой кастрюле массой 800 г от 20

    0С до кипения.

  1. В фарфоровую чашку массой 100 г при температуре 200С влили 200 г кипятка. Окончательная температура оказалась 930С. Определить удельную теплоемкость фарфора.

  1. Какое количество теплоты выделится при охлаждении стальной болванки массой 200 кг от 8000С до 150С?

Контрольная работа

по теме «Расчет количества теплоты при теплообмене»

Вариант______________

  1. Какое количество теплоты необходимо для нагревания от 20

    0С до 11200С медной детали массой 30 кг?

  1. Какое количество воды можно нагреть от 150С до кипения, затратив 714 кДж теплоты?

  1. Какое количество теплоты потребуется для нагревания 200 г спирта от 180С до 480С в стеклянной колбе массой 50 г?

  1. Сколько нужно влить холодной воды при температуре 100С в 50 кг кипятка для получения смеси с температурой 450С?

Контрольная работа

по теме «Расчет количества теплоты при теплообмене»

Вариант______________

  1. Сколько теплоты требуется для нагревания свинцовой детали массой 200 г на 700С?

  1. Сколько граммов воды можно нагреть от 00С до 1000С, сообщив ей 1200 Дж теплоты?

  1. Какое количество теплоты потребуется для нагревания смеси, состоящей из 500 г воды и 100 г спирта, от 200С до 600С?

  1. В калориметре смешали 0,39 кг воды при температуре 200С и 0,21 кг воды при температуре 600С. Определите установившуюся температуру. Потерями тепла пренебречь.

Контрольная работа

по теме «Расчет количества теплоты при теплообмене»

Вариант______________

  1. Какое количество теплоты потребуется для нагревания кирпичной печи массой 1,5 т от 100С до 200С?

  1. На сколько градусов охладится 40 кг льда, если отнять от него 400 кДж теплоты?

  1. В медной кастрюле массой 1,6 кг находится 2,3 кг воды. Сколько теплоты потребуется для нагревания от 100С до 1000С кастрюли с водой?

  1. Воду с температурой 200

    С смешивают с водой при 1000С. Определите отношение массы холодной воды к массе горячей, если установившаяся температура равна 400С. Потерями тепла пренебречь.

Консультация по физике (8 класс) по теме: Контрольная работа Количество теплоты _8кл

Вариант№2

  1. Железо массой 4 кг остывает от  до . Какая энергия при этом выделится?
  2. При сжигании каменного угля выделяется 270МДж энергии. Определить массу сгораемого угля.
  3. Сколько необходимо (массы) каменного угля для того, чтобы нагреть воду массой 2,9 кг от до ? Вариант ответа: а) 3мг б) 36г в) 2,5кг  г) 5 кг

4)Какое количество теплоты необходимо сообщить чугуну массой 250 г, чтобы его  расплавить. Начальная температура чугуна  (–20 )?

Варианты ответов: а) 2,5654МДж  б) 345,6МДж  в) 242965 Дж  г) 34МДж

Вариант№3

  1. Если использовать  228кДж энергии, то на сколько можно нагреть цинк массой 5кг? (Считать, что нагрев происходит только в твердой фазе)
  2. При кристаллизации этого вещества массой 24кг выделяется 2,88МДж.

Как, по-вашему, что это за вещество? (узнать, что это за вещество можно вычислив его удельную теплоту плавления).

  1. Для остывания жидкого цинка массой 5 кг, находящегося при температуре плавления, его заливают  водой массой 10кг, находящегося при комнатной температуре . До какой температуры остынет цинк?

Вариант ответов: а) 50,2  б) 82,3  в) 98,1  г) 50,97

  1. При производстве напыления золотой пылью на стекло, взяли 300 г  золотого пара находящегося при  температуре 2947 . Определить  энергию выделившеюся  при остывании золота до твердого состояния 1063.

Вариант№4

  1. Сколько энергии необходимо затратить, для того чтобы нагреть куб льда на , куб имеет ребро . Лед находится при .
  2. Сколько энергии выделится при сжигании спирта объемом 3 л. Плотность спирта 800 кг/м3.
  3. На спиртовую горелку положили 3 кг серебра. Если учесть, что начальная температура серебра , притом, что сжигается 20г спирта. Подсчитайте, до какой температуры нагреется этот метал? (Для простоты решения, считать, что вся энергия горючего идет на нагрев  серебра)

Вариант ответа: а) 100,2  б) 350,3  в) 901,2  г) 802,6

  1. Для производства алюминиевых ложек решили расплавить 2000 г металла. Сколько энергии необходимо затратить, если начальная температура  металла 20 ?

Варианты ответов: а) 2МДж  б) 1953920Дж  в) 5653654 МДж  г) 5,56МДж

Вариант№5

  1. На нагрев воды, массой 2кг затрачивают  672кДж энергии. Начальная температура воды . До какой температуры прогреется вода?
  2. В печке сожгли торф объемом , при этом выделилось 8,4МДж энергии. Определить плотность торфа?
  3. Сжигают 20кг нефти. Ели считать, что вся энергии нефти ушла бы, только на плавления льда. То тогда, сколько бы килограмм льда можно было бы расплавить? (Учесть, что лед находится при ).

Вариант ответов: а) 2588 кг  б) 3,56т  в)102г  г) 2678кг

Тест по физике Расчет количества теплоты 8 класс

Тест по физике Расчет количества теплоты, необходимого для нагревания тела или выделяемого им при охлаждении для учащихся 8 класса с ответами. Тест состоит из 10 заданий и предназначен для проверки знаний к главе Тепловые явления.

1. Если на нагревание 1,5 кг воды на 10 °С потребовалось 63 кДж, то какое количество теплоты будет необходимо для нагревания на то же число градусов 7,5 кг воды?

1) 31,5 кДж
2) 315 кДж
3) 840 Дж
4) 75 кДж

2. При остывании на 15 °С тело потеряло количество теплоты, равное 2500 Дж. Сколько теплоты оно потеряет, остывая на 45 °С?

1) 75 Дж
2) 750 Дж
3) 7500 Дж
4) 75 кДж

3. Медный и стальной шары массой по 0,5 кг, находившиеся при комнатной температуре (20 °С), опущены в кипяток. На нагре­вание какого из них будет затрачено большее количество те­плоты? Во сколько раз?

1) Стального; в 1,25 раза
2) Медного; в 1,25 раза
3) Стального; в 1,5 раза
4) Медного; в 1,5 раза

4. По какой формуле рассчитывают количество теплоты, которое необходимо передать телу для его нагревания и которое оно передает окружающим телам при остывании?

1) F = gph
2) Q = cm(t2 – t1)
3) F = gpV

5. Вычислите количество теплоты, необходимое для того, чтобы повысить температуру стального бруска массой 7 кг от ком­натной (20 °С) до 140 °С.

1) 42 кДж
2) 49 кДж
3) 490 кДж
4) 420 кДж

6. Каким количеством теплоты можно нагреть медный стержень массой 0,3 кг на 50 °С?

1) 600 Дж
2) 6000 Дж
3) 60 000 Дж
4) Среди ответов нет верного

7. Чугунная плита массой 100 кг, нагревшаяся на солнце до 80 °С, оказавшись в тени, остыла до 20 °С. Какое количество теплоты выделилось при этом?

1) 324 кДж
2) 32,4 кДж
3) 3240 кДж
4) 32 400 кДж

8. В алюминиевой кастрюле массой 200 г нагрели 1,2 кг воды от 20 °С до 70 °С. Какое количество теплоты пошло на это?

1) 252 кДж
2) 242,8 кДж
3) 275,2 кДж
4) 261,2 кДж

9. Сколько воды удастся нагреть на 60 °С, сообщив ей 504 кДж?

1) 20 кг
2) 2 кг
3) 200 г

10. Когда в бак горячей воды массой 10 кг с температурой 90 °С налили холодную воду с температурой 10 °С и, перемешав, из­мерили температуру, она оказалась равной 35 °С. Сколько хо­лодной воды было налито в бак?

1) 22 кг
2) 36 кг
3) ≈ 25,7 кг
4) ≈ 16 кг

Ответы на тест по физике Расчет количества теплоты, необходимого для нагревания тела или выделяемого им при охлаждении
1-2
2-3
3-1
4-2
5-4
6-1
7-2
8-3
9-2
10-1

Тест по физике Количество теплоты для 8 класса

Тест по физике Количество теплоты для 8 класса с ответами. Тест включает в себя 2 варианта, в каждом варианте 7 заданий с выбором ответа.

1 вариант

A1. Количеством теплоты называется

1) температура, которую теряет или получает тело при теплопередаче
2) внутренняя и механическая энергии тела
3) энергия, которую теряет или получает тело при теплопередаче
4) энергия, которую теряет или получает тело при движении

А2. Количество теплоты, которое необходимо для нагре­вания тела, не зависит

1) от массы этого тела
2) от способа нагрева тела
3) от изменения температуры
4) от рода вещества

А3. Удельная теплоёмкость вещества измеряется

1) в джоулях
2) в Дж/кг
3) в Дж/(кг · °С)
4) в (Дж · °С)/кг

А4. Имеются два тела из одинакового вещества. Масса первого тела больше в 2 раза массы второго тела. Если второму телу сообщить в 2 раза большее количество тепла, чем первому, то

1) второе тело нагреется в 4 раза сильнее
2) первое тело нагреется в 2 раза сильнее
3) второе тело нагреется в 2 раза сильнее
4) первое тело нагреется в 4 раза сильнее

А5. Льдине сообщили количество теплоты 42 000 Дж, в результате чего она нагрелась на 10 °С. Удельная теп­лоёмкость льда 2100 Дж/(кг · °С). Масса льдины равна

1) 0,5 кг
2) 1 кг
3) 1,5 кг
4) 2 кг

А6. У дельной теплотой сгорания топлива называется

1) количество теплоты, выделяющееся при остывании или нагревании 1 кг топлива на 1 °С
2) количество теплоты, выделяющееся при остывании всего топлива на 1 °С
3) количество теплоты, выделяющееся при полном сгорании 1 кг топлива
4) количество теплоты, выделяющееся при полном

А7. При полном сгорании бензина массой 2 кг с удельной теплотой сгорания 4,6·104 кДж/кг выделится энергия в количестве

1) 2,3 · 107 Дж
2) 29,2 · 107 Дж
3) 9,2 · 104 Дж
4) 2,3 · 104 Дж

2 вариант

A1. Удельной теплоёмкостью вещества называется

1) энергия, которую теряет или получает тело при теплопередаче
2) количество теплоты, которое необходимо передать телу массой 1 кг, чтобы изменить его температуру на 1 °С
3) температура, на которую изменится внутренняя энер­гия тела массой 1 кг при выделении энергии 1 Дж
4) количество теплоты, которое необходимо передать телу массой 1 кг, чтобы изменить его внутреннюю энергию на 1 Дж

А2. Количество теплоты, выделяемое при остывании те­ла, не зависит

1) от массы и состава вещества тела
2) от изменения температуры и окружающих условий
3) от начальной температуры и состава вещества тела
4) от потенциальной энергии тела как целого

А3. Количество теплоты измеряется

1) только в джоулях
2) только в калориях
3) в Дж/(кг · °С)
4) как в джоулях, так и в калориях

А4. Имеются два тела с разной массой из одинакового вещества. Если им сообщить равное количества тепла, то

1) тело с меньшей массой нагреется сильнее
2) тело с большей массой нагреется сильнее
3) тела нагреются одинаково сильно
4) тела нагреются произвольным образом

А5. Для нагревания стальной детали массой 400 г от 20 °С до 1500 °С необходимо количество теплоты (удельная теплоёмкость стали 500 Дж/(кг · °С))

1) 6,5 Дж
2) 1,3 кДж
3) 296 кДж
4) 26 000 кДж

А6. Выделяющаяся при сгорании топлива энергия обра­зуется за счёт

1) увеличения скорости движения молекул
2) уменьшения скорости движения молекул
3) соединения атомов в молекулы
4) разделения молекул на атомы

А7. При сгорании угля выделилось 136 · 106 Дж теплоты, удельная теплота сгорания угля 3,4 · 104 кДж/кг. Масса сгоревшего угля равна

1) 2 кг
2) 4 кг
3) 6 кг
4) 8 кг

Ответы на тест по физике Количество теплоты для 8 класса
1 вариант
А1-3
А2-2
А3-3
А4-1
А5-4
А6-3
А7-2
2 вариант
А1-2
А2-4
А3-4
А4-1
А5-3
А6-3
А7-2

Тест по физике Количество теплоты 8 класс

Тест по физике Количество теплоты Единицы количества теплоты для учащихся 8 класса с ответами. Тест состоит из 11 вопросов и предназначен для проверки знаний к главе Тепловые явления.

1. Количество теплоты — это

1) изменение внутренней энергии при излучении
2) энергия, которую тело получает или отдает при теплопере­даче
3) работа, которая совершается при нагревании тела
4) энергия, получаемая телом при нагревании

2. Количество теплоты зависит от

1) массы тела
2) того, на сколько градусов изменилась его температура
3) вещества, из которого оно состоит
4) всех этих причин

3. В каком случае телу передано большее количество теплоты, когда его нагрели от 0 °С до 10 °С (№ 1), от 10 °С до 20 °С (№ 2), от 20 °С до 30 °С (№ 3)?

1) № 1
2) № 2
3) № 3
4) Количества теплоты одинаковы

4. В каком из этих одинаковых сосудов вода нагреется до са­мой высокой температуры, если ее начальная температу­ра одна и та же и сосуды получают равные количества те­плоты?

1) № 1
2) № 2
3) № 3

5. Количество теплоты измеряют в

1) джоулях
2) ваттах
3) калориях
4) паскалях

6. Выразите количества теплоты, равные 6000 Дж и 10 000 кал, в килоджоулях.

1) 6 кДж и 4,2 кДж
2) 60 кДж и 42 кДж
3) 6 кДж и 42 кДж
4) 60 кДж и 4,2 кДж

7. Переведите количества теплоты, равные 7,5 кДж и 25 кал, в джоули.

1) 750 Дж и 10,5 Дж
2) 7500 Дж и 105 Дж
3) 750 Дж и 105 Дж
4) 7500 Дж и 10,5 Дж

8. Чтобы нагреть чашку воды, потребовалось количество тепло­ты, равное 600 Дж. На сколько и как изменилась внутренняя энергия воды?

1) На 600 Дж; уменьшилась
2) На 300 Дж; увеличилась
3) На 300 Дж; уменьшилась
4) На 600 Дж; увеличилась

9. При нагревании воды ей передано 400 Дж энергии. Какое ко­личество теплоты выделится при ее охлаждении до первона­чальной температуры?

1) 100 Дж
2) 200 Дж
3) 400 Дж
4) Для ответа нужны дополнительные данные

10. Одинаковые шары нагреты до указанных на рисунке темпе­ратур. Какому из них надо сообщить наименьшее количество теплоты, чтобы довести температуру до 300 °С?

1) № 1
2) № 2
3) № 3

11. В кастрюлю с кипятком положили взятый из холодильника, где температура 5 °С, кусок мяса. Спустя некоторое время температура воды стала равной 80 °С. Если предположить, что мясо получило при этом количество теплоты 100 Дж, то какое количество теплоты отдала ему вода?

1) Определить нельзя, так как неизвестны массы мяса и воды
2) Оно равно нулю, так как температуры воды и мяса стали одинаковыми
3) 100 Дж
4) Больше 100 Дж

Ответы на тест по физике Количество теплоты Единицы количества теплоты
1-2
2-4
3-4
4-2
5-1, 3
6-3
7-2
8-4
9-3
10-1
11-3

Калькулятор BMR

Калькулятор базовой скорости метаболизма (BMR) оценивает вашу базальную скорость метаболизма — количество энергии, израсходованной в состоянии покоя в нейтрально умеренной среде и в постабсорбтивном состоянии (это означает, что пищеварительная система неактивна, что требует примерно 12 часов голодания).

Результат

BMR = 1605 Калорий в день

Ежедневная потребность в калориях в зависимости от уровня активности

Уровень активности Калорий
Сидячий образ жизни: мало или совсем не упражнения 1,926
Упражнение 1- 3 раза в неделю 2,207
Упражнения 4-5 раз в неделю 2,351
Ежедневные упражнения или интенсивные упражнения 3–4 раза в неделю 2,488
Интенсивные упражнения 6-7 раз / неделя 2,769
Ежедневные очень интенсивные упражнения или физическая работа 3,050

Упражнение: 15-30 минут повышенной активности пульса.
Интенсивные упражнения: 45–120 минут повышенной активности пульса.
Очень интенсивные упражнения: 2+ часа повышенной активности пульса.


Калькулятор телесного жира | Калькулятор калорий

Базальный уровень метаболизма (BMR) — это количество энергии, необходимое для отдыха в умеренном климате, когда пищеварительная система неактивна. Это эквивалентно выяснению, сколько бензина потребляет неработающий автомобиль, когда он припаркован. В таком состоянии энергия будет использоваться только для поддержания жизненно важных органов, включая сердце, легкие, почки, нервную систему, кишечник, печень, легкие, половые органы, мышцы и кожу.Для большинства людей около 70% общей энергии (калорий), сжигаемой каждый день, приходится на содержание. Физическая активность составляет ~ 20% расходов, а ~ 10% используется для переваривания пищи, также известного как термогенез.

BMR измеряется в очень строгих условиях в состоянии бодрствования. Для точного измерения BMR необходимо, чтобы симпатическая нервная система человека была неактивна, что означает, что человек должен быть полностью отдохнувшим. Основной обмен веществ обычно является самым большим компонентом общей потребности человека в калориях. Суточная потребность в калориях — это значение BMR, умноженное на коэффициент от 1,2 до 1,9, в зависимости от уровня активности.

В большинстве случаев BMR оценивается с помощью уравнений, полученных на основе статистических данных. Уравнение Харриса-Бенедикта было одним из первых введенных уравнений. В 1984 г. оно было пересмотрено для большей точности и использовалось до 1990 г., когда было введено уравнение Миффлина-Сент-Джера. Уравнение Миффлина-Сент-Джера оказалось более точным, чем пересмотренное уравнение Харриса-Бенедикта.Формула Кэтча-МакАрдла немного отличается тем, что рассчитывает дневные затраты энергии в состоянии покоя (RDEE) с учетом безжировой массы тела, чего не делают ни Миффлин-Сент-Джор, ни уравнение Харриса-Бенедикта. Из этих уравнений наиболее точным уравнением для расчета BMR считается уравнение Миффлина-Сент-Джера, за исключением того, что формула Кетча-МакАрдла может быть более точной для людей, которые стройнее и знают процентное содержание жира в организме. Вы можете выбрать уравнение, которое будет использоваться в расчетах, развернув настройки.

Три уравнения, используемые калькулятором, перечислены ниже:

Уравнение Mifflin-St Jeor:

Для мужчин:

BMR = 10Вт + 6.25H — 5A + 5

Для женщин:

BMR = 10 Вт + 6,25 ч — 5A — 161

Пересмотренное уравнение Харриса-Бенедикта:

Для мужчин:

BMR = 13,397 Вт + 4,799 ч — 5,677A + 88,362

Для женщин:

BMR = 9,247 Вт + 3,098 ч — 4,330 A + 447,593

Формула Кэтча-Макардла:

BMR = 370 + 21.6 (1 — F) ш

где:

W — масса тела, кг
H — рост в см
Возраст
F — телесный жир в процентах

Переменные BMR

Muscle Mass — Аэробные упражнения, такие как бег или езда на велосипеде, не влияют на BMR. Однако анаэробные упражнения, такие как поднятие тяжестей, косвенно приводят к более высокому BMR, поскольку они наращивают мышечную массу, увеличивая потребление энергии в состоянии покоя. Чем больше мышечной массы в физическом составе человека, тем выше BMR требуется для поддержания его тела на определенном уровне.

Возраст — Чем старше и гибче человек, тем ниже его BMR или тем ниже минимальное потребление калорий, необходимое для поддержания функционирования его органов на определенном уровне.

Генетика — Наследственные черты, переданные от предков, влияют на BMR.

Weather — Холодная среда повышает BMR из-за энергии, необходимой для создания гомеостатической температуры тела. Точно так же слишком много внешнего тепла может повысить BMR, поскольку тело расходует энергию на охлаждение внутренних органов.BMR увеличивается примерно на 7% с каждым увеличением внутренней температуры тела на 1,36 градуса по Фаренгейту.

Диета — Небольшие, обычно распределенные порции пищи увеличивают BMR. С другой стороны, голодание может снизить BMR на 30%. Подобно телефону, который переходит в режим энергосбережения в течение последних 5% заряда батареи, человеческое тело будет приносить жертвы, такие как уровень энергии, настроение, поддержание физического состояния и функций мозга, чтобы более эффективно использовать то небольшое количество калорий. энергия используется для его поддержания.

Беременность — Обеспечение существования отдельного плода изнутри увеличивает BMR. Вот почему беременные женщины едят больше обычного. Кроме того, менопауза может увеличивать или уменьшать BMR в зависимости от гормональных изменений.

Добавки — Некоторые добавки или лекарства повышают BMR, в основном, чтобы способствовать снижению веса. Кофеин — обычное дело.

BMR Тесты

Онлайн-тесты BMR с жесткими формулами — не самый точный метод определения BMR человека.Лучше проконсультироваться у сертифицированного специалиста или измерить BMR калориметрическим прибором. Эти портативные устройства доступны во многих клубах здоровья и фитнеса, кабинетах врачей и клиниках похудания.

Скорость метаболизма в покое

Хотя эти два понятия используются как синонимы, между их определениями есть ключевое различие. Скорость метаболизма в состоянии покоя или сокращенно RMR — это скорость, с которой организм сжигает энергию в расслабленном, но не полностью неактивном состоянии. Его также иногда определяют как расход энергии в состоянии покоя или РЗЭ.Измерения BMR должны соответствовать общему физиологическому равновесию, в то время как условия измерения RMR могут быть изменены и определены контекстными ограничениями.

Современная мудрость

Проведенное в 2005 году метааналитическое исследование BMR * показало, что при контроле всех факторов скорости метаболизма между людьми все еще существует неизвестная разница в 26%. По сути, средний человек, соблюдающий среднюю диету, вероятно, будет иметь ожидаемые значения BMR, но есть факторы, которые все еще не поняты, которые точно определяют BMR.

Следовательно, все расчеты BMR, даже с использованием самых точных методов, проводимых специалистами, не будут абсолютно точными в их измерениях. Еще не все функции человеческого тела хорошо изучены, поэтому расчет общего суточного расхода энергии (TDEE), полученный из оценок BMR, является всего лишь оценкой. При работе над достижением каких-либо целей в области здоровья или фитнеса BMR может помочь заложить основы, но с этого момента ему больше нечего предложить. Расчетный BMR и, следовательно, TDEE могут привести к неудовлетворительным результатам из-за их приблизительных оценок, но ведения ежедневного журнала упражнений, потребления пищи и т. Д., может помочь отследить факторы, которые приводят к каким-либо результатам, и помочь определить, что работает, а также что необходимо улучшить. Отслеживание прогресса в указанном журнале и внесение изменений с течением времени по мере необходимости, как правило, является лучшим показателем прогресса в достижении личных целей.

Номер ссылки

* Johnstone AM, Murison SD, Duncan JS, Rance KA, Speakman JR, Факторы, влияющие на изменение базальной скорости метаболизма, включают массу без жира, массу жира, возраст и циркулирующий тироксин, но не пол, циркулирующий лептин или трийодтиронин1.Am J Clin Nutr 2005; 82: 941-948.

Страница не найдена | MIT

Перейти к содержанию ↓
  • Образование
  • Исследование
  • Инновации
  • Прием + помощь
  • Студенческая жизнь
  • Новости
  • Выпускников
  • О MIT
  • Подробнее ↓
    • Прием + помощь
    • Студенческая жизнь
    • Новости
    • Выпускников
    • О MIT
Меню ↓ Поиск Меню Ой, похоже, мы не смогли найти то, что вы искали!
Попробуйте поискать что-нибудь еще! Что вы ищете? Увидеть больше результатов

Предложения или отзывы?

тепловой энергии, необходимой для превращения льда в пар, Рон Куртус

SfC Home> Физика> Тепловая энергия>

, автор: Рон Куртус (от 17 января 2019 г.)

Чтобы превратить лед в воду и воду в пар, требуется определенное количество тепловой энергии или тепловой энергии.

При нагревании материала вы добавляете тепловую кинетическую энергию его молекулам и обычно повышаете его температуру. Единственное исключение — когда материал достигает точки плавления или кипения. При этих двух температурах тепловая энергия переходит в изменение состояния или фазы материала. После изменения состояния температура снова повысится с добавлением тепловой энергии.

Скорость изменения температуры равна удельной теплоемкости материала. Количество энергии, необходимое для плавления материала, называется скрытой теплотой плавления .Все это можно проиллюстрировать, показав, сколько тепла требуется, чтобы превратить лед в воду, а затем превратить воду в пар.

Вопросы, которые могут у вас возникнуть:

  • Какие единицы измерения используются при превращении льда в пар?
  • Как используется удельная теплоемкость?
  • Как используется скрытое тепло?

Этот урок ответит на эти вопросы. Полезный инструмент: Конвертация единиц



Единицы измерения

Поскольку мы измеряем количество тепла, необходимое для внесения этих изменений, нам необходимо знать определения различных задействованных единиц.

Теплопередача

Тепло — это полная кинетическая энергия всех молекул в объекте. Хотя энергия обычно измеряется в джоулях , более распространенной единицей измерения тепла является калорий , которая определяется как количество тепла, необходимое для изменения температуры 1 грамма воды на 1 градус Цельсия. Существует переводной коэффициент для соотношения джоулей и калорий, но мы не будем об этом беспокоиться.

Примечание : калории сокращаются как кал , а граммы — как г .Кроме того, o C означает градусы Цельсия.

В английской системе измерения используется BTU (британская тепловая единица), которая представляет собой количество тепла, необходимое для изменения температуры 1 фунта воды на 1 градус по Фаренгейту. BTU рассматривается в Соединенных Штатах, когда относится к мощности печи.

Удельная теплоемкость

Материалы различаются по способности накапливать тепловую энергию. Например, такой материал, как железо, нагревается намного быстрее, чем вода или дерево.Измерение количества тепла, необходимого для изменения температуры единицы массы вещества на 1 градус, называется его удельной теплоемкостью .

Имеются таблицы с указанием удельной теплоемкости различных материалов. В приведенной ниже таблице показана удельная теплоемкость льда, воды и пара в единицах калорий на грамм-градус Цельсия.

(Обратите внимание, что все элементы перечислены с одинаковым количеством десятичных знаков. Это указывает на одинаковую точность для каждого. Кроме того, ноль перед десятичной точкой гарантирует, что читатель будет знать, что это десятичная точка, а не муха. .)

Состояние воды
Удельная теплоемкость, кал / г- o C

Лед

0,50

Вода

1,00

Пар

0,48

Удельная теплоемкость воды в различных состояниях

Другими словами, для нагревания воды на один градус потребуется в два раза больше калорий, чем для нагрева той же массы льда на один градус.

Скрытое тепло

Когда какой-либо материал нагревается до температуры, при которой он меняет состояние, температура остается неизменной до тех пор, пока не изменится состояние всего материала. Это означает, что температура ледяной воды будет оставаться при 0 o C (32 o F) до тех пор, пока весь лед не растает. То же самое и при охлаждении материалов.

Причина в том, что для изменения состояния с твердого на жидкое или с жидкого на газ необходимо затратить энергию. Точно так же необходимо отводить энергию для изменения состояния при охлаждении материала.Требуемое количество энергии называется скрытой теплотой замораживания или кипения. На диаграмме ниже показано скрытое тепло или энергия, необходимая для изменения состояния воды.

Изменение состояния воды
Скрытое тепло, кал / г

Плавление / замораживание

80

Кипение / конденсация

540

Скрытое тепло, необходимое для изменения состояния воды

Проблема

Хороший способ понять концепции — решить проблему.

Предположим, у нас есть 50 г льда при температуре -10 o C. Мы хотим нагреть материал до тех пор, пока он не превратится в пар при 110 o C. Сколько тепла требуется?

Для такой сложной проблемы, как эта, хорошо бы разбить ее на части и решить каждую часть индивидуально. Это также помогает объяснить логику, использованную в решении.

1. Нагревание льда

Сколько тепла потребуется, чтобы поднять 50 г льда до точки плавления?

Температура льда должна быть увеличена на 10 градусов до 0 o C.

Поскольку удельная теплоемкость льда составляет 0,50 кал / г — o ° C, это означает, что для выращивания 1 г льда требуется 0,50 калорий 1 o ° C. Таким образом, для повышения температуры на 50 г требуется 50 x 0,50 калорий 1. o C и 10 x 50 x 0,50 = 250 кал , чтобы поднять лед до точки плавления.

2. Тающий лед

Сколько тепла потребуется, чтобы растопить 50 г льда?

Скрытая теплота таяния льда составляет 80 кал / г. Это означает, что для таяния 1 г льда требуется 80 калорий тепла.

Таким образом, для плавления 50 г требуется 50 x 80 = 4000 кал .

3. Отопительная вода

Сколько тепла требуется, чтобы нагреть 50 г воды с 0 o C до точки кипения 100 o C?

Поскольку удельная теплоемкость воды составляет 1,00 кал / г — o ° C, это означает, что для получения 1 г воды 1 o ° C необходимо 1,00 калории. Таким образом, для повышения уровня на 50 г требуется 50 x 1,00 калорий. o C и 100 x 50 x 1,00 = 5000 кал , чтобы поднять воду до точки кипения.

4. Кипяток

Сколько тепла потребуется, чтобы вскипятить 50 г воды?

Скрытая теплота кипящей воды составляет 540 кал / г. Это означает, что для кипения 1 г воды требуется 540 калорий тепла.

Таким образом, для кипячения 50 г требуется 50 x 540 = 27000 кал .

5. Греющий пар

Сколько тепла требуется для нагрева 50 г пара с 100 o C до 110 90 254 o C?

Так как удельная теплота пара равна 0.48 кал / г — o C, это означает, что для повышения 1 г необходимо 0,48 калорий 1 o C. Таким образом, для повышения 50 г пара 1 o C и 10 x 50 потребуется 50 x 0,48 калорий. x 0,48 = 240 кал для повышения температуры пара до 110 o C.

6. Всего

Общее количество тепла, необходимое для превращения 50 г льда при -10 o C в пар при 110 90 254 o C, составляет:

250 + 4000 + 5000 + 27000 + 240 = 36490 кал .

Сводка

Нагревательные материалы, такие как лед, вода и пар, повышают их температуру. Удельная теплоемкость материала определяет количество калорий, необходимое для нагрева одной единицы массы на один градус.

Изменение состояния материала требует дополнительной тепловой энергии, которая не используется для изменения температуры. Количество тепла, необходимое для изменения состояния материала, называется его скрытой теплотой. Сложную задачу определения количества тепла, необходимого для превращения льда в воду и последующего превращения воды в пар, следует разбивать на части и решать индивидуально.


Разогрейте мышление


Ресурсы и ссылки

Полномочия Рона Куртуса

Сайтов

Уравнения теплоемкости — Wolfram Science World

Физические ресурсы

Книги

Книги с самым высоким рейтингом по тепловой энергии

Лучшие книги по физике температуры


Вопросы и комментарии

Есть ли у вас какие-либо вопросы, комментарии или мнения по этой теме? Если это так, отправьте свой отзыв по электронной почте.Я постараюсь вернуться к вам как можно скорее.


Поделиться страницей

Нажмите кнопку, чтобы добавить эту страницу в закладки или поделиться ею через Twitter, Facebook, электронную почту или другие службы:


Студенты и исследователи

Веб-адрес этой страницы:
www.school-for-champions.com/science/
heat_ice_steam.htm

Пожалуйста, включите это как ссылку на свой веб-сайт или как ссылку в своем отчете, документе или диссертации.

Авторские права © Ограничения


Где ты сейчас?

Школа чемпионов

По физике

Тепловая энергия, необходимая для превращения льда в пар

Уравнение химического баланса — Online Balancer

Balance Chemical Equation — Онлайн-балансир

Введите химическое уравнение для баланса:

Инструкции по балансировке химических уравнений:
  • Введите уравнение химической реакции и нажмите «Баланс».Ответ появится под
  • Всегда используйте верхний регистр для первого символа в имени элемента и нижний регистр для второго символа. Примеры: Fe, Au, Co, Br, C, O, N, F. Сравните: Co — кобальт и CO — монооксид углерода
  • Чтобы ввести электрон в химическое уравнение, используйте {-} или e
  • Чтобы ввести ион укажите заряд после соединения в фигурных скобках: {+3}, {3+} или {3}.
    Пример: Fe {3+} + I {-} = Fe {2+} + I2
  • Заменить неизменяемые группы в химических соединениях, чтобы избежать двусмысленности.
    Например, уравнение C6H5C2H5 + O2 = C6H5OH + CO2 + h3O не будет сбалансировано,
    , но PhC2H5 + O2 = PhOH + CO2 + h3O будет
  • Состояния соединения [например, (s) (aq) или (g)] не требуются .
  • Если вы не знаете, какие продукты входят в состав, введите только реагенты и нажмите «Баланс». Во многих случаях полное уравнение будет предложено.
  • Стехиометрию реакции можно вычислить для сбалансированного уравнения. Введите количество молей или вес для одного из соединений, чтобы вычислить остальное.
  • Ограничивающий реагент можно вычислить по сбалансированному уравнению, введя количество молей или вес для всех реагентов.
Примеры полных химических уравнений для баланса:
  • Fe + Cl 2 = FeCl 3
  • KMnO 4 + HCl = KCl + MnCl 2 + H 2 O + Cl 2
  • K 4 Fe (CN) 6 + H 2 SO 4 + H 2 O = K 2 SO 4 + FeSO 4 + (NH 4 ) 2 SO 4 + CO
  • C 6 H 5 COOH + O 2 = CO 2 + H 2 O
  • K 4 Fe (CN) 6 + KMnO 4 + H 2 SO 4 = KHSO 4 + Fe 2 (SO 4 ) 3 + MnSO 4 + HNO 3 + CO 2 + H 2 O
  • Cr 2 O 7 {-2} + H {+} + {-} = Cr {+3} + H 2 O
  • S {-2} + I 2 = I {-} + S
  • PhCH 3 9060 0 + KMnO 4 + H 2 SO 4 = PhCOOH + K 2 SO 4 + MnSO 4 + H 2 O
  • CuSO 4 * 5H 2 O = CuSO 4 + H 2 O
  • гидроксид кальция + диоксид углерода = карбонат кальция + вода
  • сера + озон = диоксид серы
Примеры реагентов химических уравнений (будет предложено полное уравнение):
  • H 2 SO 4 + K 4 Fe (CN) 6 + KMnO 4
  • Ca (OH) 2 + H 3 PO 4
  • Na 2 S 2 O 3 + I 2
  • C 8 H 18 + O 2
  • водород + кислород
  • пропан + кислород
Поделитесь с нами своим мнением о своем опыте использования балансировщика химических уравнений.

химическое уравнение сбалансировано сегодня

Вернуться в меню химических инструментов в Интернете
Используя этот веб-сайт, вы подтверждаете свое согласие с Положениями и условиями и Политикой конфиденциальности.
© 2020 webqc.org Все права защищены

Физические демонстрации — тепло

Физические демонстрации — тепло Хотя на первый взгляд кажется, что тепло не имеет ничего общего с движением, оно теперь понял, что тепло — это движение молекул.Связь между тепло и движение были созданы Бенджамином Томпсоном (1753-1814), Американец кто симпатизировал британцам во время войны за независимость и в итоге поселился в Баварии и стал графом Рамфордом [1]. Во время просмотра скучный артиллерийских орудий в Мюнхене в 1798 году, он пришел к выводу, что движение может превратиться в тепло и оценить механический эквивалент высокая температура. * Спустя полвека эксперимент был уточнен и повторен Джеймсом. Прескотт Джоуль (1818–1889), сын английского пивовара.Джоуль осторожен количественный измерения стали образцом для современной науки [2]. Для наших целей мы будет включать в главу, посвященную теплу, такие темы, как механика жидкости, фаза переходы, криогеника, термодинамика, кинетическая теория и горение.

* Теперь определяется как 4,186 джоулей на калорию.

ССЫЛКИ
1. С. К. Браун, Бенджамин Томпсон, граф Рамфорд , MIT Press: Кембридж, Массачусетс (1979).

2. Х. Дж. Стеффенс, Джеймс Прескотт Джоуль и концепция энергии , Публикации истории науки: Нью-Йорк (1979).


Таблица 2.1

Физические свойства выбранных газов

 Плотность кипения при молекулярном замораживании 
Точка веса (г / литр)
Формула газа (г / моль) (° C) (° C) (1 атм)

Аргон Ar 39.95-189,2 -185,9 1,633
Двуокись углерода CO2 44,01 -78,5a 1,799
Гелий He 4,00 -272,2b -268,9 0,164
Водород h3 2,02 -259,1 -252,9 0,082
Метан Ch5 16,04 -182,6 -161,5 0,656
Neon Ne 20,18 -248,7 - 246,1 0,825
Азот N2 28,01 -209,9 -195,8 1,145
Кислород O2 32,00 -218.4 -183,0 1,308
Гексафторид серы SF6 146,05 -63,7a 5,970
Ксенон Xe 131,30 -112,0 -108,1 5,367

a Точка сублимации.
b При 26 атмосфер. 4 Он ниже не замерзнет давление около 20 атмосфер.

Общие правила техники безопасности

Демонстрации в этой главе представляют особую опасность, так как большинство связаны с очень горячими или очень холодными веществами, летучими химическими веществами, хрупкий стеклянная посуда, газы под высоким давлением или откачанные емкости.Процедуры должен быть разработанным, чтобы гарантировать, что демонстрация может быть проведена без Опасность публике или самому себе. Горячие и холодные вещества должны быть обработанный щипцами или специальными термоизолированными перчатками. Защита глаз следует носить не только во избежание травм, но и для правильной установки пример для публики. Огнетушитель, аптечка, аварийный душ и рядом должен быть телефон. * Человек, исполняющий демонстрации должны знать, где они расположены и как использовать их.Помощник, обученный оказанию первой помощи и СЛР, обеспечивает дополнительную меру охрана. Если в аудитории есть дети, их следует предупрежден вообще не пытаться проводить некоторые демонстрации и не делать другие если не присутствует родитель или другой взрослый. Причина предосторожность им следует объяснить.

* Можно получить в отделе безопасности лаборатории, P.O. Box 1368, Джейнсвилл, WI 53547-1368 (608) 754-2354

При ожоге обожженную кожу положить под холод. вода пока не пройдет боль.Если волдыри присутствуют, они должны быть покрытый стерильной марлей. Если ожог серьезный или поражена большая площадь участвует, нужно обратиться к врачу. При обморожении место следует утеплить медленно с теплой (около 106 ° F) водой или другой частью тела, например, положить палец под подмышку. Порезы и ссадины должны быть промывают мыльной водой, просушивают и накрывают стерильной марлей. Если кровотечение тяжелый, надавите на порез и / или точку пульса над порез.Поднимите разрез выше уровня сердца. Если кровотечение продолжается или если порез серьезный, следует обратиться к врачу в течение нескольких часов, чтобы в вырезать прошитый.

Меры безопасности, относящиеся к звуку, электричеству и лазерам, будут будет рассмотрено в следующих главах.


2,1

Пушка с жидким азотом

Быстрое испарение жидкого азота внутри стального баллона оказывает давление, достаточное для того, чтобы сдуть пробку с цилиндра.
МАТЕРИАЛЫ
  • Дьюар жидкого азота
  • стальной цилиндр с внутренней чашкой меньшего размера из нержавеющей стали
  • пробка
  • молоток
ПРОЦЕДУРА
Захватывающая демонстрация испарения жидкости азот можно сделать, опустив небольшую чашку из нержавеющей стали (~ 30 миллилитров) приостановлено проволокой и заполнен жидким азотом в больший стальной цилиндр (40 см в длину × 5 см и.d.) запечатанный с одного конца. Пробка — это толченый в другой конец молотком. Потом забирают все устройство а также встряхивают, чтобы из чашки вылился азот. Когда жидкость азот ударяется о стенку цилиндра, он быстро испаряется, вызывая большой и внезапное повышение давления, которое с большой силой срывает пробку и шум. Тяжелое основание цилиндра помогает поглотить удары в отдача. Жидкий азот можно приобрести в больницах.Если Сосуд Дьюара отсутствует, можно использовать обычный термос для хранения азота, если он сделан из металла или стекла Pyrex. Обыкновенный стекло может разбиться при низкой температуре жидкого азота, а емкость не должна быть плотно закрыта, иначе термос бутылка взорвется, когда азот испарится. Если жидкий азот не имеется в наличии, вместо него можно использовать кусок сухого льда [1].
ОБСУЖДЕНИЕ
Когда жидкость испаряется, образующийся газ занимает гораздо больше объем чем жидкость, из которой он появился, если давление одинаково в обоих случаи.Расширение обычно составляет примерно 1000 раз для самый жидкости (сравните плотность газа с плотностью жидкости откуда оно пришло). Если газ не расширяется, давление увеличится во столько же раз. Повышенное давление повышает кипячение точка и тормозит закипание, как в скороварке, но недостаточно чтобы предотвратить сильное повышение давления, как показано здесь. в жидкость азотная пушка, требуется лишь небольшая часть жидкого азота закипятить, чтобы со значительным усилием сдувать пробку.
ОПАСНОСТИ
Жидкий азот кипит при температуре -196 ° C (-321 ° F, 77K). и может вызвать сильное обморожение. Следует соблюдать осторожность, чтобы избежать контакта с участием жидкость и все, что было охлаждено ею. Помимо изготовление убедитесь, что цилиндр достаточно прочный, единственная другая мера предосторожности — прицелиться пушка над головами публики. Пробка может проехать несколько сто футов, но относительно безвреден после того, как отскочит от стены или потолок.
ССЫЛКА
1. Дж. С. Миллер, Physics Fun and Demonstrations , Central Научный Компания: Чикаго (1974).

2,2

Сворачивающаяся банка

Небольшое количество воды наливается в алюминиевую банку для безалкогольных напитков и привел до кипения над горелкой Бунзена, а затем перевернуть в ванну с холодом вода, заставляя банку мгновенно разрушиться в результате быстрого конденсация пара.
МАТЕРИАЛЫ
  • алюминий, банка для безалкогольных напитков
  • 15 мл воды
  • Горелка Бунзена или плита
  • щипцы большие
  • поднос с холодной водой
ПРОЦЕДУРА
Налейте около 15 миллилитров воды в пустой алюминиевый корпус объемом 12 унций. безалкогольный напиток банку (хорошо, если в названии есть «Crush») и нагрейте ее по Бунзену горелка или нагревайте плиту, пока облако конденсированного водяного пара не выйдет из рот банки примерно на 20 секунд.Следует отметить, что облако не «пар», который представляет собой невидимый газ или дым, который является скоплением из крошечные твердые частицы. Используя щипцы, быстро снимите банку с горелки. и переверните его в поддон с холодной водой на глубину нескольких сантиметры. Банка мгновенно разрушится [1,2].
ОБСУЖДЕНИЕ
Когда банка остывает, водяной пар конденсируется, снижая давление. внутри банка, из которой была удалена большая часть воздуха.Площадь поверхности из банки составляет около 0,031 м 2 (или 48 из 2 ). в самый крайний случай, перепад давления в 1 атмосферу (14,7 фунта / дюйм 2 ) приложил бы общую силу около 700 фунтов, что намного превышает что может выдержать банка. Можно было предположить, что вода быть втянут в банку. Однако, поскольку отверстие в банке небольшое а также конденсация происходит быстро, вязкость воды препятствует этому от входа в банку.Некоторое количество воды попадает в банку, и это эта вода, которая заставляет пар так быстро конденсироваться.
ОПАСНОСТИ
В банке кипит вода с температурой 100 ° C, и она может вызвать ожог кожа при прикосновении к ней незащищенными руками. Убедитесь, что снаружи в банка чистая, чтобы предотвратить возгорание.
ССЫЛКИ
1. Кауфман Г., Journ. College Sci. Учить. 14 , 364 (1985).

2. Шахашири Б.З., Химические демонстрации , г. Университет of Wisconsin Press: Madison, Wisconsin, Vol 2 (1985).


2,3

Желоб для двуокиси углерода

Углекислый газ из стеклянного стакана выливают в желоб, содержащий количество свечей, которые последовательно гаснут невидимым газ.
МАТЕРИАЛЫ
  • V-образный желоб с установленным внутри количеством свечей *
  • Стеклянный стакан 2 л
  • картонная крышка для стакана
  • небольшой блок сухого льда
  • щипцы
  • матчи
  • конфорка (опция)
* Доступен в компании Frey Scientific
ПРОЦЕДУРА
V-образный желоб из металла или оргстекла, наклоненный под углом примерно 30 ° содержит около пяти свечей, установленных вертикально внутри желоба с их фитили ниже верхней части желоба.Свечи зажигаются в честь чей-то день рождения. Есть большая вероятность, что кто-то в аудитория будет день рождения в любой день, если будет не менее 250 человек в аудитории (см. таблицу 2.2). 2-литровый стеклянный стакан, содержащий углерод газообразный диоксид удаляется из-за лекционного стола и сливается в впадина. Свечи гаснут одна за другой, как если бы желоб, но стакан явно пустой, так как углекислый газ бесцветный, газ без запаха.

Углекислый газ получали, помещая кусок сухого льда в стакан перед лекцией с куском картона сверху, чтобы закрыть стакан. Картон удаляется, и сухой лед вынимается. с участием щипцы непосредственно перед тем, как стакан будет показан публике. Стакан может быть хранить на слегка теплой электрической плите, чтобы предотвратить образование из водяного льда на его внешней стороне и для обеспечения быстрого испарения сухой лед. Сухой лед часто можно купить у продавцов мороженого.

ОБСУЖДЕНИЕ
Эта демонстрация иллюстрирует необходимость кислорода для поддержания горение. Он также демонстрирует сублимацию, процесс, при котором жидкость изменяется непосредственно из твердого тела в газ, минуя жидкую фазу. Сухой лед называют «сухим», потому что, в отличие от обычного водяного льда, он сублимирует при атмосферном давлении. Обыкновенные сублиматы льда (а значит, и сухой) когда давление пара воды над жидкостью ниже 4.58 торр (или 0,006 атмосферы). Это состояние может существовать в засушливый день и в причина, по которой снег и лед в конечном итоге исчезают, даже когда температура ниже точки замерзания (0 ° C). Жидкая двуокись углерода не может существовать ниже в давление тройной точки 5,1 атмосфер и температура -56,2 ° С. Углекислый газ остается в стакане, потому что его примерно на 50% больше. плотный чем воздух. В некоторых типах огнетушителей используется углекислый газ.
ОПАСНОСТИ
Сухой лед возгоняется при температуре -78.5 ° C и может вызвать обморожение. Таким образом нельзя допускать касания голой кожи.

Таблица 2.2

Вероятность того, что у кого-то из аудитории данного размера будет День рождения в заданный день

 Вероятность размера аудитории 
10 3%
20 5%
30 8%
70 13%
100 24%
150 34%
200 42%
250 50%
300 56%
400 67%
500 75%

2.4

Взрывающиеся мыльные пузыри

Мыльные пузыри, выдутые природным газом или водородом, воспламеняются с помощью свеча когда они поднимаются к потолку.
МАТЕРИАЛЫ
  • раствор мыльного пузыря
  • стеклянная труба
  • природный газ (метан) или водород
  • свеча
  • матчи
ПРОЦЕДУРА
Мыльные пузыри можно сделать, наполнив стеклянную трубку небольшим количеством мыльный раствор (продается в магазинах игрушек).Также могут быть найдены подходящие решения добавив несколько капель олеата натрия или калия или жидкую посуду моющее средство в стакан с теплой, дистиллированной или очень мягкой чистой водой. Небольшое количество глицерина и нескольких капель аммиачной воды значительно улучшает прочный качество пузырей [1].

труба поддерживается кольцевой стойкой и соединяется с источником сжатого газа. Газ легче воздуха, такой как гелий, * позволяет пузырькам подняться к потолку по мере выхода из трубы.Газ должен дать возможность ненадолго протечь по трубе, прежде чем заполнять ее мылом раствор, чтобы изгнать воздух. Если пузыри надуваются водород их можно зажечь, держа в руке свечу, когда они поднимаются [2,3]. Некоторые кислород можно смешать с водородом, используя схему, показанную на в фигура, чтобы сделать взрыв гораздо громче. Самый громкий взрыв будет происходить когда отношение водорода к кислороду имеет стехиометрическое значение 2: 1, поэтому для получения чистого H 2 O.Природный газ, часто удобно имеется в наличии на скамейке для лекций горит очень красивое и тихое пламя. Натуральный газ состоит в основном из метана (CH 4 ), который немного меньше плотный чем воздух. Смеси метана и кислорода поднимутся, только если соотношение CH 4 — O 2 большой. Эффект лучше всего, если смотреть на темном фоне в приглушенном свете.

* Специальные газы и оборудование можно приобрести в Мэтисон Газовые продукты, П.О. Box 1587, Secaucus, NJ 07094.

ОБСУЖДЕНИЕ
Эта демонстрация иллюстрирует ряд физических концепций. Очень существование пузырей — яркая демонстрация поверхностного натяжения. А пузырек состоит из тонкой пленки жидкости, удерживаемой поверхностью напряжение. Он имеет эластичные свойства, аналогичные свойствам воздушного шара. Размер в пузырек определяется давлением газа внутри него и радиус и толщина водной пленки.Что газ внутри пузыря находится под давление можно проиллюстрировать, надув трубкой пузырь, но перед пузырек выпускается, позволяя ему сдуваться и задуть свечу около шток трубы. Меньший пузырь имеет более высокое давление, чем больше пузырь по той же причине, что воздушный шар сначала трудно надуть но становится легче по мере увеличения. Это можно проиллюстрировать с помощью Т-образный труба, в которой выдуваются два пузыря разного размера, а затем связано вместе посредством трубы.Более крупный пузырь станет больше, а меньший пузырек станет меньше.

Тенденция пузыря подниматься, когда он наполнен газом легче воздух иллюстрирует принцип Архимеда. Пузырь поднимется, если его вес плюс вес газа внутри меньше веса воздуха смещен. При комнатной температуре и атмосферном давлении кубический метр воздуха весит почти три фунта. Пузырь, наполненный воздухом, медленно опустится на пол из-за веса пленки воды.Вес воздуха внутри приблизительно компенсирует вес вытесненного воздуха, пренебрегая маленький перепад давления. От радиуса пузыря и его конца скорости при падении (см. обсуждение в разделе 1.4), можно оценить вес воды и, следовательно, толщина водной пленки ( плотность воды 1000 кг / м3 3 ). Заполненный воздухом пузырь может быть заставить плавать в ванне с углекислым газом, произведенным сухим льдом в дно прозрачного контейнера, например, аквариума.

Пузыри очень увлекательны и поучительны. Целые лекции могут быть даны о свойствах и поведении пузырей и т. демонстрации [4-7] неизбежно увлекают публику.

ОПАСНОСТИ
Помимо обычных мер предосторожности, связанных с пламенем, кислород смеси может произвести довольно громкий взрыв. Демонстратору следует использовать беруши. и предупредить присутствующих, чтобы они держали руки над своими уши.
ССЫЛКИ
1. С. Л. Стронг, Scientific American 220 , 128 (май 1969 г.).

2. Шахашири Б.З., Химические демонстрации , г. Университет of Wisconsin Press: Madison, Wisconsin, Vol 1 (1983).

3. Х. А. Робинсон, изд., Демонстрации лекций по физике , Американец Институт физики: Нью-Йорк (1963).

4. К. В. Мальчики, Мыльные пузыри и силы, которые их формируют , Educational Services Incorporated, Doubleday Anchor Books: Garden City, Нью-Йорк (1959).

5. К. Изенберг, Наука о мыльных пленках и мыльных пузырях , Tiesto Лтд .: Клеведон (1978).

6. С. Саймон, Магия мыльных пузырей , Латроп, Ли и Шепард: Новый Йорк (1985).

7. А. Уорд, Эксперименты с поверхностным натяжением и пузырьками , Дриада Пресса: Лондон (1985).


2,5

Двигатель героя

Специально сконструированная стеклянная колба с водой и взвешенными веществами. из вверху цепочкой быстро вращается при нагревании снизу.
МАТЕРИАЛЫ
  • колба с паровыми трубками (двигатель Героя) *
  • Горелка Бунзена
  • матчи
  • вода
* Можно приобрести в компании Carolina Biological Supply Company, Central Научная компания, Эдмунд Сайентифик, Фишер Сайентифик, Фрей Научный Company и Sargent-Welch Scientific Company
ПРОЦЕДУРА
Стеклянная колба со стеклянными трубками, соединенными с горлышком и согнутыми в одну. боковая сторона, подвешенный на цепочке из бисера, содержащий небольшое количество воды и нагретый снизу горелкой Бунзена, быстро вращается, когда вода фурункул.Стеклянные трубки действуют как струи, выпускающие пар за один направление. Цепочка из бусинок обеспечивает свободу вращения, поскольку не скручивается. В количество использованной воды не критично. Следует использовать достаточно, чтобы не выкипает, но если использовать слишком много, это займет много времени время закипеть. Скорость вращения можно регулировать, изменяя количество тепла, подаваемого снизу. Более прочная версия может быть построен с помощью алюминиевого цилиндра [1].Устройство называется «Двигатель Героя». после греческий философ Герой Александрии, который в I веке до н. э. написал об этом, но, видимо, так и не сделал. Аналогия могла бы к вращающемуся оросителю газона, что также можно продемонстрировать на лекционный зал, использующий сжатый воздух вместо воды.
ОБСУЖДЕНИЕ
Эта демонстрация иллюстрирует сохранение углового момента, Ньютона третий закон (действие и противодействие) и принцип ракетного двигателя.В качестве водяной пар вытесняется, он передает равный и противоположный импульс к колбе, заставляя ее вращаться в противоположном направлении. Это иллюстрирует преобразование тепловой энергии в механическую и показывает большое количество пара, которое может быть произведено из небольшого количества вода. Обратите внимание на большую простоту этого движка по сравнению с другими формами паровые машины, у которых много сложных движущихся частей. Возможно это было бы быть более широко используемым, если бы не тот факт, что он чрезвычайно неэффективный при преобразовании тепла в движение, потому что скорость истечения его относительно низкое и много тепла теряется (выбрасывается в воздух).
ОПАСНОСТИ
Колба должна быть такого типа, который рассчитан на нагрев горелка. Следует следить за тем, чтобы вода не закипела слишком быстро, иначе колба начать бесконтрольно крутиться. Пламя должно быть потушено до вода полностью закипело. Колбе необходимо дать остыть перед тем, как тронут незащищенными руками.
ССЫЛКА
1. L. Hirsch, Am. Journ. Phys. 46 , 773 (1978).

2.6

Модель Гейзер

Модель газовой колонки, состоящая из водяного столба, нагреваемого снизу с помощью Горелка Бунзена периодически вспыхивает и стреляет водой до потолка.
МАТЕРИАЛЫ
  • модель гейзер
  • Горелка Бунзена
ПРОЦЕДУРА
Работающую модель гейзера построить достаточно просто [1]. Это требует только длинная вертикальная заполненная водой трубка с сужением в верхняя, несущая конструкция, горелка Бунзена для обогрева трубы снизу и улавливающий бассейн для сбора воды и возврата ее в трубку после извержение вулкана.В зависимости от длины и объема трубки и количества тепла используемый, он может извергаться каждые несколько минут. Водяной столб около 2 метры высота работает хорошо, поскольку она делает точку кипения примерно на 5 ° C выше внизу, чем вверху. Если трубка сделана из стекла пирекс, можно наблюдать активность во всей трубе гейзера. Можно противопоставить турбулентный характер вскипания до регулярного периодического характера высыпания.
ОБСУЖДЕНИЕ
Гейзер работает, нагревая воду в нижней части трубы до перегретое состояние (выше точки кипения воды при атмосферный давление).Однако вода закипает не сразу, так как кипячение точка повышается давлением водяного столба выше. Когда это наконец-то закипает, давление водяного столба сбрасывается, и в начинается бурное кипение, вызывающее сыпь. Выброшенная вода охлаждает при контакте с воздухом и уловителем и стекает обратно вниз трубка и процесс нагрева начинается снова.

Помимо извержения, должна быть возможность наблюдать тремор различных типов, а также эффект гидроудара, возникающий сразу после извержение, когда вода на мгновение не может стекать трубка давлением пара, остающегося в трубке.Поскольку пар конденсируется, в трубке гейзера образуется частичный вакуум и слышно «отстой» бассейн с водой из уловителя в трубу с большой силой. Результирующий шум звучит как отрыжка и неизменно вызывает реакция из зала. Аналогичный эффект можно получить, если налить воду. в воронку, вставленную через резиновую пробку в пустую колбу.

ОПАСНОСТИ
Гейзер относительно безопасен, если проявлять обычную осторожность, а не трогательный горелку Бунзена или трубку с горячей водой.Вода может причина горит, если подойти слишком близко при извержении, но к тому времени, когда вода имеет разлетелся в воздух и упал обратно, это не более опасно, чем горячий душ. Одна вода может не попасть в уловитель и повредить другую. рядом, поблизости оборудование.
ССЫЛКА
1. Л. В. Андерсон, Дж. В. Андерегг и Дж. Э. Лоулер, Американский журнал из Наука 278 , 725 (1978).

2,7

Магдебургские полушария

Два полушария при их соединении и эвакуации не могут быть вытянуты Кроме из-за атмосферного давления.
МАТЕРИАЛЫ
  • Магдебургские полушария *
  • вакуумный насос
  • манометр (опция)
* Можно получить в Central Scientific Company, Fisher Научный и Frey Scientific Company
ПРОЦЕДУРА
Ярко продемонстрирована большая сила, которую может оказывать атмосфера. с участием пара полушарий Магдебурга, названная в честь города Магдебург в Германия где Отто фон Герике (1602-1686), физик и политик, а мэр города в 1657 году развлекал местных жителей двумя команды восьми лошадей каждая пытается разорвать два медных полуметра диаметром полушария удерживаются вместе только давлением атмосферы [1].Отто фон Герике изобрел один из первых вакуумных насосов примерно в 1641 году и был в значительной степени ответственным за развенчание средневекового представления о том, что «природа ненавидит вакуум «.

Для более скромной демонстрации можно использовать стальные полусферы с диаметром около 12 сантиметров и какой-то прокладкой для предотвращения утечка воздуха вокруг стыка. Если хороший вакуумный насос не имеется в наличии, можно использовать полусферы большего размера и водяной аспиратор или ручной насос для производить частичный вакуум.У одного из полушарий должна быть губа вокруг край чтобы полусферы не скользили по прокладке. В полушария размещаются вместе и откачиваются вакуумным насосом через шланг связано к клапану на стороне одного из полушарий. Затем клапан закрыть и шланг снят. Полушария обычно снабжены ручками. чтобы двое добровольцев из аудитории могли попытаться развести их. Это безопасно предложить им денежную сумму, если они добьются успеха (не открывая в клапан!).

В варианте демонстрации можно использовать два диска с та же площадь проекции, что и полушария, чтобы показать, что объем эвакуирован пространство не имеет значения. Резиновые присоски различных типов можно показать. Известный пример — вантуз. Пара таких плунжеры можно использовать как недорогой вариант магдебургского полушарие демонстрация.

ОБСУЖДЕНИЕ
Атмосфера оказывает силу (пи) r 2 p или 6.25 (пи) кв дюйм × 14,7 фунта / кв. Дюйм = 290 фунтов для полной сферы диаметром 5 дюймов эвакуированы. Даже при частичном вакууме почти невозможно два человека нормальной силы, чтобы разделить их. Весы можно вешать из одно из полушарий, а другое прикреплено к надежному крючку в потолок, чтобы получить количественную оценку атмосферного давления. С участием Полусферы Отто фон Герике, сила более двух тонн!
ОПАСНОСТИ
Предполагая, что сферы сделаны из материала, достаточно прочного, чтобы быть раздавленным атмосферой, единственная опасность исходит от одного из волонтеры падение, если другой будет слишком сильно тянуть или отпустить.Подходящий вывод к демонстрация заключается в том, чтобы открыть клапан и впустить воздух, пока волонтеры все еще тянут, и в этом случае им следует дать указание не тянуть слишком сложно.
ССЫЛКА
1. J. Higbie, Am. Journ. Phys. 48 , 987 (1980).

2,8

Невозможный воздушный шар

Кажется, что специально сконструированный воздушный шар обладает большой подъемной силой. за пределами что разрешено принципом Архимеда.
МАТЕРИАЛЫ
  • специально сконструированный воздушный шар
  • вес
ПРОЦЕДУРА
Профессор Эд Миллер из Висконсинского университета любит исполнять демонстрация вовлекающий мошенничество с плавучестью, в котором воздушный шар диаметром полметра против потолок, под которым висит груз. После объяснения всего о плавучести, профессор Миллер с энтузиазмом рассказывает о захватывающе разработка некоторыми из его коллег нового газа с 47-кратным увеличением подъем мощность водорода и как нам очень повезло добиться прогресса демонстрация.Затем он вешает на веревку еще несколько, очевидно, тяжелых грузов. дно воздушного шара. В конце концов кто-то просыпается и пытается спорить что это несовместимо со всем, что он сказал. Он не говорит слово, просто поворачивается, протягивает руку и вытаскивает пробку, чтобы освободить в газ из баллона. Воздушный шар «выдувает» свой газ и опускается. ни к чему, но веса остаются висеть там. Очевидно, есть строка через воздушный шар к небольшому крючку в потолке, который удерживает веса вверх.Наконец кольцо на вершине шара, которое удерживало его, падает. вниз вокруг рухнувшего шара на гири с грохотом, как юмористический кульминация аферы.
ОБСУЖДЕНИЕ
Вес, который может поднять воздушный шар, равен весу воздух, вытесненный воздушным шаром, за вычетом веса воздушного шара и в содержащийся в нем газ (принцип Архимеда). Таким образом, водород (H 2 ) с молекулярной массой 2 грамма на моль по сравнению с воздухом с в среднем молекулярная масса 29 грамм на моль близка к лучшей, что можно делать.Даже газ с нулевой молекулярной массой (вакуум) может обеспечить лифт всего на 7% больше, чем у водорода. Использование эвакуированного баллона является одним из курс, непрактично, так как воздушный шар был бы слишком тяжелым, если бы был сделан из материал достаточно прочный, чтобы выдерживать атмосферное давление. В давление в баллоне, наполненном водородом, на самом деле немного больше, чем давление окружающей атмосферы (обычно 5-10%), и поэтому такой воздушный шар будет содержать больше молей водорода, чем вытесняемый воздух, который немного снижает его плавучесть.
ОПАСНОСТИ
Единственная опасность в этой демонстрации — разрыв струны и падение вес на ноге.

2,9

Варка со льдом

Воду в закрытой колбе доводят до кипения, переворачивая колбу и держа кубик льда на его дне.
МАТЕРИАЛЫ
  • Колба кипяченая, толстостенная, круглодонная, 1 л,
  • резиновая пробка на два отверстия
  • термометр
  • стеклянная трубка
  • резиновый шланг с зажимом
  • Горелка Бунзена
  • кубик льда
ПРОЦЕДУРА
Воду можно довести до кипения при температуре значительно ниже 100 ° C с помощью сокращение давление атмосферы над водой [1-5].1-литровый, толстостенный, Круглая колба для кипячения примерно наполовину наполняется водой. А два отверстия резиновая пробка используется для герметизации колбы. Термометр помещается в один отверстие пробки. В другом отверстии находится стеклянная трубка, подключенная к резинка шланг, который можно закрыть зажимом. Вода подается в варить на горелке Бунзена. Затем воде дают остыть в течение одного минута. Затем зажимной зажим используется для герметизации колбы, колбу переворачивают, а к его дну прижимается кубик льда.Вода в колбе будет кипятить несколько секунд.
ОБСУЖДЕНИЕ
Температура, при которой жидкость закипает, зависит от давления в атмосфера над ним. Вода закипит, когда давление ее пара (см. Таблица 2.3) превышает давление окружающего газа. В этом демонстрация давление понижается от нормального атмосферного давления (760 торр) на охлаждение воздуха в колбе и конденсация водяного пара в колбе.По мере закипания воды давление в колбе увеличивается. к точка, в которой оно больше, чем давление пара жидкости, а также вода перестает кипеть. Колба должна быть заполнена не более чем наполовину. вода, чтобы обеспечить пространство для пара, образующегося при кипячении. Если слишком мало пространство, давление пара жидкой воды может быть достиг до кипения.
ОПАСНОСТИ
Помимо опасности ожога от горелки Бунзена и горячего колба колба может взорваться, если давление будет достаточно снижено.Этот рисковать сводится к минимуму за счет использования толстостенной колбы для кипячения, а не стандарт круглодонный.
ССЫЛКИ
1. Д. Бэйсли, Учитель естествознания 47 , 45 (май 1980 г.).

2. А. Джозеф, П. Ф. Брандвейн, Э. Морхольт, Х. Поллак и Дж. Ф. Кастка, Справочник по физическим наукам , Harcourt, Brace and Мир: Нью-Йорк (1961).

3. Х. А. Робинсон, изд., Демонстрации лекций по физике , Американец Институт физики: Нью-Йорк (1963).

4. Шахашири Б.З., Химические демонстрации , г. Университет of Wisconsin Press: Madison, Wisconsin, Vol 2 (1985).

5. J. P. VanCleave, Teaching the Fun of Physics , Prentice Зал Пресса: Нью-Йорк (1985).


Таблица 2.3

Температурная зависимость давления пара воды

 Температура (° C) Давление (торр) 
0 4.579
5 6,543
10 9,209
15 12,788
20 17,535
25 23,756
30 31,824
35 42,175
40 55,324
45 71,88
50 92.51
55 118,04
60 149,38
65 187,54
70 233,7
75 289,1
80 355,1
85 433,6
90 525,76
95 633,90
100 760.00

2,10

Замораживание испарением

Вода комнатной температуры в колбе бурно закипает, а затем перевертывается. в лед при понижении давления в колбе.
МАТЕРИАЛЫ
  • колба толстостенная круглодонная
  • резиновый вакуумный шланг с клапаном
  • ловушка для серной кислоты
  • вакуумный насос
  • вода дистиллированная
  • манометр (опция)
ПРОЦЕДУРА
С помощью механического вакуумного насоса можно сделать воду в колбе. кипятить так сильно при комнатной температуре, что потеря тепла из-за испарение вызывает замерзание воды [1].Аппарат для такого криофор демонстрация может быть куплена или построена. должен используйте толстую колбу с круглым дном и ловушку, которую можно наполнить с сильной серной кислотой, чтобы изолировать колбу от вакуумного насоса. В кислота поглощает водяной пар, тем самым увеличивая скорость кипения. а также предотвращает загрязнение масла насоса водой. В рекомендуется использование дистиллированной воды. Воду можно подавать в кипятить быстрее, если его предварительно нагреть.Большой манометр, видимый для аудитория — полезное дополнение.

Эту демонстрацию лучше всего начать, задав вопрос аудитории, на каком температура воды закипит. Большинство людей ответят с температурой 212 ° F или 100 ° С. Следует отметить, что этот ответ верен только при атмосферный давление (760 торр), и это хорошо известно жителям большая высота города, что вареное яйцо там дольше готовят, потому что вода кипит при более низкой температуре на большей высоте.Неточно сказать что там яйцо варится дольше, потому что вода на самом деле привел до кипения быстрее. Барометрическое давление падает примерно на 3% за каждый 1000 футов над уровнем моря. Такой город, как Денвер, штат Колорадо, находится в высота около 5000 футов, и, таким образом, среднее барометрическое давление составляет около 660 торр, а температура кипения воды около 96 ° C. На вершине Mt. Эверест (29 028 футов) заварить чай — тяжелая работа.Можно указать из того, что кипящая жидкость имеет тенденцию поддерживать постоянную температуру, и Вот почему многие продукты готовят в кипящей воде. Под давлением плита, точка кипения повышается за счет повышенного давления.

One затем включает вакуумный насос и позволяет аудитории смотреть, как вода бешено закипает. Можно дотронуться до колбы и указать, что Это совсем не жарко, и, вообще-то, даже немного прохладно. Пока взявшись за фляжку, спросите публику, при какой температуре вода замерзнет.Если это правильно рассчитано, вода замерзнет по сигналу. в густая форма льда. В большом зале лед лучше всего виден в силуэт на экране, освещенном дуговой лампой или диапроектором.

ОБСУЖДЕНИЕ
Теплота испарения воды составляет 540 калорий на грамм, а высокая температура плавления воды составляет 80 калорий на грамм. Плотность воды 1 грамм на миллилитр. Когда вода испаряется, выделяется большое количество тепла. таким образом извлекается из воды, вызывая охлаждение.Испарение каждого миллилитр воды способен заморозить почти семь миллилитров один раз температура воды снизилась до 0 ° C. Поскольку температура падает, давление пара воды падает, и кипение сильно уменьшено. Из-за этой потери тепла мы чувствуем прохладу, когда выходим из ванны или бассейн. Животные потеют, чтобы сохранять хладнокровие. процесс. Если бы наш пот испарился слишком быстро, мы бы замерзли!
ОПАСНОСТИ
Основная опасность заключается в взрыве колбы, если она не предназначена для выдерживать атмосферного давления или если его уронили или ударили тяжелым предметом.Серная кислота также может вызвать ожоги, и с ней следует обращаться осторожно. осторожность.
ССЫЛКА
1. Х. А. Робинсон, изд., Демонстрационные лекции по физике , Американец Институт физики: Нью-Йорк (1963).

2,11

Негорючий платок

Хлопковый платок обливают жидкостью и поджигают, но носовой платок не горит.
МАТЕРИАЛЫ
  • платок из хлопка или долларовая купюра
  • 50% раствор изопропилового спирта и воды
  • небольшое количество соли (хлорид натрия)
  • спички или прикуриватель
  • щипцы длиной не менее 30 см
  • ведро воды (рекомендуется)
  • монета и сигарета (по желанию)
ПРОЦЕДУРА
Из кармана достают ярко окрашенный хлопковый платок или лучше Тем не менее, доброволец, посаженный в зал и пропитанный жидкостью и подожгли [1].Держа платок щипцами и наблюдая синее пламя указывает на то, что платок не горит. в заключение носовой платок выдувают быстрым рывком или обливают ведром воды и передали для проверки или вернули добровольцу. Для большего драматизма помощник может потушить пламя огнем. огнетушитель. Волонтера можно попросить поднять носовой платок, чтобы все увидеть.

В этой демонстрации использованная жидкость представляет собой смесь 50/50 изопропил спирт и вода.Метанол или этанол можно заменить на изопропил алкоголь. Немного соли в растворе поможет сделать пламя более сильным. видимый. Другие горючие материалы, такие как бумага, также могут быть использованы вместо из платок [2]. Долларовая банкнота, предоставленная кем-то из аудитории особенно эффективен. Обратите внимание на чистоту спирта. перед смешивание. Медицинский спирт, продаваемый в аптеках, часто составляет 30% и более. вода. Можно повторить демонстрацию с различными смесями алкоголя и вода.Слишком много воды предотвратит возгорание спирта, а также маленький вода позволит ткани или бумаге обугливаться.

В варианте демонстрации оборачивают сухую хлопчатобумажную ткань. плотно вокруг монеты, и зажженная сигарета касается ткани. В монета поглощает тепло и поддерживает температуру ниже точки, в которой ткань ожоги.

ОБСУЖДЕНИЕ
Эта демонстрация иллюстрирует изменение температуры требуется для поддержки горения в различных веществах.Спирт горит на температура ниже температуры растопки хлопка. Кроме того, отопление испарение воды отводит тепло и предотвращает повреждение ткани. жжение.
ОПАСНОСТИ
Хотя пламя относительно холодное, оно способно производить тяжелая форма ожоги. Возьмите платок очень длинными щипцами (30 см или Больше) пока он воспламеняется. Планируйте заранее, чтобы успеть его погасить. в вода полностью испарилась или случайно упала на пол во время горения.Изопропиловый спирт может серьезно повредить глаза.
ССЫЛКИ
1. Б.З. Шахашири, Chemical Demonstrations , Университет из Wisconsin Press: Madison, Wisconsin, Vol 1 (1983).

2. J. Jardin, P. Murray, J. Tyszka, J. Czarnecki, Journ. из Химическая Образование 55 , 655 (1978).


2,12

Облако жидкого азота

Жидкий азот вызывает быстрое испарение путем вытеснения его из большой Дьюар под давлением охлаждает воздух и вызывает образование большого, плотное облако.
МАТЕРИАЛЫ
  • 25-литровый сосуд Дьюара с жидким азотом
  • Резиновая пробка с 2 отверстиями для установки Dewar
  • медная труба специальной конструкции с выхлопными отверстиями
  • сжатый воздух или азот
  • электрические нагревательные ленты (опция)
ПРОЦЕДУРА
Впечатляющее облако можно получить при быстром кипении жидкости. азот. Подходящий аппарат состоит из большого (25-литрового) Дьюара, заполненного жидкость азот.Резиновая пробка с двумя отверстиями закрывает отверстие сосуда Дьюара. В один В отверстие пробки помещается трубка, соединенная с воздухом высокого давления или через регулятор давления в баллон со сжатым азотом. В в в другое отверстие помещается трубка, ведущая к центру метровой длины, толстостенный, горизонтальная, медная труба, запаянная на концах, с дюжиной и более мелких Отверстия диаметром 1/8 дюйма в верхней части, через которые может выходить азот. в комнате. Электрические нагревательные ленты вокруг трубы могут быть используемый для прогрева трубы перед использованием.Если труба с достаточно толстыми стенками отсутствует, внутри может свободно помещаться цельный медный цилиндр в труба. Цель состоит в том, чтобы обеспечить теплое отверстие с высокой теплоемкостью для причина быстрое кипение азота как можно дольше. В зависимости от в теплоемкость трубы и расход газа в сосуд Дьюара, а облако подниматься с пола на несколько метров вверх можно в течение хорошей доли минуты, пока труба не остынет до слишком низкого температура.Демонстрацию нельзя повторять около часа, потому что в труба должна прогреться до комнатной температуры.

Устройство работает с нагнетанием жидкого азота. из Дьюара в теплую трубу, где он быстро закипает и выходит из в отверстия в трубе как холодный азот. Затем холодный газ конденсируется. влажность из воздуха в помещении над аппаратом и образует облако. Таким образом демонстрация лучше всего работает при высокой влажности.Если есть вентиляция системы в комнате, ее можно выключить за несколько минут до демонстрация чтобы позволить влажности подняться. Примерно через 30 секунд труба будет прохладно до точки, где из отверстий бьют фонтаны жидкости. В жидкость не так эффективна, как холодный газ, для охлаждения воздуха, а также облако медленно спадает. Цветные огни, освещающие облако обеспечивает дополнительная визуальная привлекательность. Демонстрация — эффективное завершение презентация, а демонстратор может исчезнуть в облаке и возможно выйти из комнаты через ближайшую дверь, невидимую для зрителей.

ОБСУЖДЕНИЕ
Хотя эта демонстрация проводится в основном для драмы, ряд физический показаны принципы, в том числе закон Паскаля, теплота испарение теплоемкость, перенос тепла и конденсация водяного пара в воздух при понижении температуры. Механизм похож на способ облака в небе образуются за счет охлаждения воздуха до температуры ниже который воздух насыщается влагой (относительная влажность 100%).В облако состоит из очень маленьких капель жидкой воды и не является паром или курите, столько людей ответят, если их спросят.
ОПАСНОСТИ
Возможная опасность — взрыв сосуда Дьюара из-за избыточного давления. Выходные отверстия не должны быть заблокированы, а давление должно превышать около 30 фунтов на квадратный дюйм не следует использовать, если не известно, что сосуд Дьюара прочность. Целесообразно испытать Дьюара за подходящей баррикадой под давлением примерно на 50% больше, чем предполагается использовать.Немного жидкого азота воля выход из отверстий, особенно после того, как устройство поработало некоторое время, а также таким образом, чтобы избежать обморожения, лицо и другие части тела должны не быть прямо над отверстиями в трубе, когда она включена.

2,13

Передатчик тепла

Спичка в фокусе параболического отражателя зажигается от радиация от электрического нагревательного элемента, размещенного в фокусе второй параболический отражатель поперек комнаты и направлен на первую.
МАТЕРИАЛЫ
  • 2 параболических отражателя *
  • электрический нагревательный элемент
  • деревянная спичка или порох
  • лист бумаги (для выравнивания)
* Можно получить в компании Central Scientific, Эдмунд Научный, Fisher Scientific, Frey Scientific Company и Sargent-Welch Scientific Компания
ПРОЦЕДУРА
Передача тепла излучением может быть эффективно продемонстрирована с помощью два параболических отражателя предпочтительно диаметром 20 сантиметров или более.Рядом с фокусом одного рефлектора размещается резистивный электрический резистор. обогрев элемент. В фокусе другого отражателя помещается деревянная спичка. Затем отражатели размещаются на расстоянии нескольких метров друг от друга и нацелены на один еще один. Когда на нагревательный элемент подается напряжение, спичка должна разорваться. пламя за несколько секунд. Можно показать, что стоять в луче безопасно, поскольку поток мощности (ватт на квадратный метр) довольно низкий. Отражатели нужно не иметь оптического качества, так как источник тепла довольно сильно разбросан, но поверхности должны отражать инфракрасное излучение с высокой эффективностью.Хром обшивка отражателей полезна. Менее эффективный вариант использует одиночный отражатель фокусирует изображение лампы на спичке [1]. А кусок бумаги можно использовать в тускло освещенной комнате, чтобы рассмотреть красное пятно, на котором тепло направляется так, чтобы спичка находилась в оптимальном месте. Избегайте натыкаясь любой из отражателей после их совмещения. Еще немного драма спичку можно заменить небольшим количеством пороха, как сэр Хамфри Дэви (1778-1829) любил выступать с публичными лекциями в Королевском дворце. Учреждение в Лондоне [2].
ОБСУЖДЕНИЕ
Излучение — это один из трех способов передачи тепла. В другие — проводимость и конвекция. Только радиация может иметь место в вакуум. Остальные требуют материальной среды. Это средство который солнце греет землю. Тепловое излучение — это форма электромагнитного волна в инфракрасной части спектра. Имеет ту же форму, что и легкий, микроволны, радиоволны, рентгеновские лучи и т. д., за исключением других длина волны.Когда температура тела увеличивается, он излучает все более короткие и более короткие длины волн, сначала выделяя тепло, а затем свет. Наиболее материалы растают до того, как они испустят значительное количество ультрафиолета. В количество энергии, излучаемой телом, пропорционально четвертой степени его абсолютная температура (закон Стефана). Инфракрасное излучение подчиняется тому же законы геометрической оптики, как и свет, и поэтому он может быть сфокусирован зеркала и линзы, как в этой демонстрации.Радиация также может быть описанный с точки зрения испускания и поглощения отдельных фотонов.
ОПАСНОСТИ
Нагревательный элемент может вызвать ожоги или поражение электрическим током при использовании. прямо от ЛЭП. Сильный ожог может возникнуть, если какая-либо часть тела или одежда помещается в фокус приемного отражателя. Не пытайся к выровняйте отражатели, поместив глаз в фокус; скорее наблюдайте в пятно на листе бумаги. Должны быть предоставлены средства для безопасного тушить спичку, когда она зажжена, и чтобы не уронить спичку на что-нибудь это может загореться.Поэкспериментируйте с небольшим количеством пороха, чтобы получить взрыв впечатляющий, но безопасный.
ССЫЛКИ
1. Р. Э. Берг, Phys. Обучайте 28 , 56 (1990).

2. Дж. Тиндаль, Тепло; Режим движения , третье издание, Лонгман: Лондон (1868 г.).


2,14

Взрыв паров этанола

Небольшое количество этанола, помещенное в бутылку, взрывается и дуть пробку на значительное расстояние с помощью электрической искры.
МАТЕРИАЛЫ
  • Флакон полиэтиленовый 250 мл с двумя гвоздями по бокам
  • пробка (или резиновая пробка) для бутылки
  • небольшое количество этанола> 95%
  • переносная катушка Тесла * или другой источник высокого напряжения
* Поставляется в Fisher Scientific и Sargent-Welch Научный Компания
ПРОЦЕДУРА
От двух до три миллилитры этанола> 95% (C 2 H 5 OH) в 250-миллилитровая полиэтиленовая бутылка с двумя гвоздями или винтами в стороны, как показано на рисунке [1].Кончики гвоздей или шурупов заострены и находятся в пределах нескольких миллиметров от касания. Бутылка является закрывается пробкой или резиновой пробкой. Флакон можно кратковременно встряхнуть. чтобы жидкий этанол пришел в равновесие с его пар. Затем гвозди подключаются к источнику высокого напряжения (> 1000 вольт), слаботочный источник. Образовавшаяся искра воспламеняет этанол и пробивает пробку. выкл. В качестве альтернативы можно поднести ручную катушку Тесла к одному из гвоздей.Даже с отключенным вторым гвоздем емкости хватает между гвозди, чтобы искра проскочила через зазор. Емкость может быть увеличенным, вызывая более интенсивную искру, подключив второй гвоздь к земле или к любому проводнику с большой площадью поверхности. Номер такой бутылки могут быть расположены последовательно электрически, в результате чего все пробки сдувать одновременно. Бутылки большего размера можно использовать для большего впечатляющий взрывы.Можно отметить, что автомобили могут использовать этанол в качестве топлива и что реакция в цилиндрах двигателя такая же, как у иллюстрированный Вот.
ОБСУЖДЕНИЕ
В этой демонстрации часть жидкого этанола испаряется и смешивается. с воздухом в бутылке. Происходит реакция C 2 H 5 OH + 3O 2 -> 2CO 2 + 3H 2 O. Таким образом, является увеличивают как количество молей газа (с 4 до 5), так и его температура в результате энергии, выделяющейся в экзотермической реакции.В выросла давление заставляет пробку сдувать бутылку. Демонстрация не можем повторить, не промывая бутылку, потому что взрыв потребляет весь кислород в баллоне.
ОПАСНОСТИ
Стопор выталкивается со значительной силой, поэтому следует соблюдать осторожность. быть приняты во избежание повреждения осветительных приборов или других предметов. Сохраняйте область Над бутылкой не должно быть препятствий. Напряжение, используемое для создания искра должна быть либо высокой частоты, как в катушке Тесла, либо слабый ток, чтобы избежать поражения электрическим током.
ССЫЛКА
1. Б.З. Шахашири, Chemical Demonstrations , Университет из Wisconsin Press: Мэдисон, Висконсин, Том 2 (1985).

2,15

Тепловая конвекция

Свеча гаснет, когда плотно прилегающий стеклянный цилиндр размещен над ним, если в цилиндр не опускается Т-образный кусок металла.
МАТЕРИАЛЫ
  • свеча
  • стеклянная трубка, длина 30 см × диаметр 3 см
  • металлический тройник
  • матчи
  • катушка медного провода (опция)
ПРОЦЕДУРА
Конвекцию нагретого воздуха можно продемонстрировать достаточно просто и эффективно со свечой [1].Свеча зажигается, чтобы показать публике, что это вполне обыкновенный. Затем стеклянный цилиндр длиной около 30 см и диаметром 3 см, открытый с обоих концов, опускается на свечу, плотно закрывая в основание, чтобы воздух не мог подниматься снизу. Через несколько секунд в свеча погаснет. Затем тонкая, плотно прилегающая перегородка из медь или другой подобный металл опускается в цилиндр до мало сантиметры верха свечи.Перегородку можно держать в место Т-образным удлинением вверху. Цилиндр поднят, свеча перезапустить, и цилиндр снова надеть свечу. Свеча продолжается гореть мерцающим пламенем, пока баллон находится на месте. В заключение, через некоторое время перегородка снимается, и свеча гаснет. За каждый случае, аудиторию можно попросить предсказать, будет ли свеча Продолжать сжечь. Немного попрактиковавшись, свечу можно отпустить. вне а затем оживить, быстро опустив перегородку.

Свечу в качестве оборки можно также погасить, резко прикоснувшись к ней. катушку с медной проволокой к пламени [2]. Медь проводит тепло прочь и снижает температуру ниже точки возгорания воска свечи.

ОБСУЖДЕНИЕ
При отсутствии перегородки поднимающийся вверх теплый воздух мешает с падающим вниз холодным воздухом в верхней части цилиндра, и в конвекционный узор не достигает уровня свечи.С участием перегородка на месте, теплый воздух поднимается вверх по одной стороне перегородки и прохладный воздух падает с другой стороны и наполняет свечу с участием кислород. Идет ли конвекция по часовой стрелке или против часовой стрелки около перегородка зависит от небольшой асимметрии, когда начинается конвекция. Как только конвекция начинается в одном направлении, она будет иметь тенденцию продолжаться. в в этом направлении, пока горит свеча, так же, как и направление в закрученная вода, выходящая из канализации в ванне, в основном зависит от маленький при выдергивании пробки в воде присутствует завихренность. *

* Сила Кориолиса из-за вращения Земли будет определить направление завихрения, если начальная завихренность достаточно маленький. Эта сила направлена ​​в противоположных направлениях в северной и южный полушарие.

Конвекция, наряду с излучением и проводимостью, является одним из способов в какое тепло переносится из одного места в другое. Конвекция может происходить в газах и жидкостях. Конвекция вызывается градиентами давления в жидкость.У свечи градиент давления небольшой, поэтому более теплый воздух менее плотный, чем более холодный воздух, и поэтому сильно притягивается к земле силой тяжести. Горение свечи требует сила тяжести как можно продемонстрировать, наблюдая, как свеча в свободном падении погасить [3].

ОПАСНОСТИ
Опасности в этой демонстрации довольно очевидны и включают ожоги. от свечи и вырезы из стекла.
ССЫЛКИ
1.Дж. С. Миллер, Physics Fun and Demonstrations , Central Научный Компания: Чикаго (1974).

2. Т. Л. Лием, Приглашения на исследование по науке , Ginn Press: Лексингтон, Массачусетс (1981).

3. Шахашири Б.З., Химические демонстрации , г. Университет of Wisconsin Press: Madison, Wisconsin, Vol 2 (1985).


2,16

Взрывающиеся шары

Воздушные шары, наполненные гелием и водородом, привязанные веревками над лекция скамейки лопаются спичкой на конце палки.
МАТЕРИАЛЫ
  • два одинаковых шара
  • 20 футов тонкой струны
  • газообразный гелий и водород
  • деревянная палка длиной около 1 метра с зажимом на одном конце
  • матчи
ПРОЦЕДУРА
Два одинаковых шара заполнены гелием, другой — гелием. водород. Воздушные шары прикрепляются к любому концу лекционной скамейки тонкими струны, которые позволяют им парить на высоте десяти футов над полом.В надувные шары должен быть заправлен в течение примерно часа после использования, так как газ будет постепенно распространятся по воздушным шарам, и они потеряют плавучесть.

Зрителей просят угадать, что находится в воздушных шарах. Большинство людей предположим, что они заполнены гелием. Затем аудиторию просят предсказать, что произойдет, когда зажженная спичка коснется одного из надувные шары. Большинство людей скажут, что воздушный шар лопнет.

Зажигается спичка и прикрепляется зажимом из кожи аллигатора на конце деревянный палка длиной около метра.Спичка касается гелиевого шара вызывая это лопнуть. Затем слушателей спрашивают, готовы ли они подобно чтобы увидеть, как это повторяется с другим шариком Затем повторяется демонстрация с водородным баллоном, вызвав большой взрыв и огненный шар.

Когда аудитория приходит в себя, можно объяснить важность ничего не принимать как должное в науке. Гелий — лишь один из примеров газ легче воздуха.На самом деле водород еще легче (примерно вдвое тяжелее). Однако восходящая выталкивающая сила незначительна. разные поскольку он дается разницей между весом газа в в баллон и вес воздуха, вытесняемого баллоном. На банке продолжить обсудить принцип Архимеда, химическое горение или научный метод. Упоминание о Гинденбурге и причинах, по которым водород больше не используется. используется в дирижаблях. Работа воздушных шаров на воздушном шаре может обсуждаться.

Демонстрация также может быть проведена с использованием других газов, таких как воздух и кислород, если воздушные шары подвешены каким-либо образом, например, с помощью кольца стоять. Также можно использовать стехиометрическую смесь (2: 1) водорода и кислорода. если приняты надлежащие меры для защиты ушей от громкого взрыв что приводит к [1]. Если газообразный водород недоступен, его можно производить в колба Эрленмейера из пирекса объемом 250 мл, в которую помещается от 10 до 15 граммов замшелого цинка а также Добавили 65 мл 3М соляной кислоты [2].Можно увеличить количество звука, возникающего при взрыве водородного шара, помещая некоторые воздух в баллоне до того, как он наполнится водородом. Экспериментируйте, чтобы получить соответствующий драматический, но не опасно громкий взрыв.

ОБСУЖДЕНИЕ
Тенденция воздушного шара подниматься при наполнении газом легче, чем воздух иллюстрирует принцип Архимеда. Воздушный шар поднимется, если его вес плюс вес газа внутри меньше веса воздуха смещен.При комнатной температуре и атмосферном давлении кубический метр воздуха весит почти три фунта. Можно сделать воздушный шар или мыльный пузырь к плавать в ванне с двуокисью углерода, образованной сухим льдом на дне прозрачный контейнер, например, аквариум.

Горение газа в воздушном шаре является иллюстрацией химический реакция, в данном случае водород в баллоне реагирует с кислородом в окружающий воздух с образованием воды с выделением 232 кДж / моль вода сформирован.Реакция идет гораздо интенсивнее, если кислород смешивается с водород, чем если бы он смешивался с кислородом воздуха. Это дает некоторое указание скорости, с которой газы диффундируют, когда перегородка между ними (воздушный шар) внезапно удаляется.

ОПАСНОСТИ
Эта демонстрация выглядит опаснее, чем есть на самом деле. Если вокруг и, особенно, над воздушным шаром, не должно быть препятствий, и если воздушный шар воспламеняется палкой длиной не менее метра на расстоянии вытянутой руки, относительно Безопасно.Чтобы обрести уверенность, нужно потренироваться с маленькими воздушными шарами. Помнить что объем водорода пропорционален кубу диаметра воздушного шара, и, таким образом, демонстрация быстро масштабируется до довольно драматический пропорции. Водородно-кислородные смеси требуют защиты органов слуха как для в демонстрант и публика.
ССЫЛКИ
1. Б.З. Шахашири, Chemical Demonstrations , Университет из Wisconsin Press: Madison, Wisconsin, Vol 1 (1983).

2. С. Исом и К. Лейл, Учитель естественных наук 56 , 45 (март 1989).


2,17

Дымовые кольца

Картонная коробка с отверстием с одной стороны используется для дыма. вихри.
МАТЕРИАЛЫ
  • картонная коробка
  • лист резины или пластика
  • источник дыма
  • прожектор (опционально)
  • свеча (по желанию)
ПРОЦЕДУРА
Удаляется одна из шести граней картонной коробки произвольного размера. а также заменяется листом тонкой резины или пластика.Пластиковый душ занавес или даже листа ткани будет достаточно. Лист приклеен к коробке, чтобы сделайте как можно более плотную посадку. Круглое отверстие около 10 см по диаметру вырезается в картоне на стороне коробки напротив простынь. Ящик наполнен дымом. Хороший способ сделать это — раскрасить ободок лунки водным раствором тетрахлорида титана (TiCl 4 ) с помощью ватного тампона. В качестве альтернативы коробка может быть заполненный дымом от сгорания какого-либо подходящего материала или, возможно, Больше безопасно, с помощью мела или талька.Если прожектор или другое яркий свет есть, он должен быть направлен в отверстие в коробке со стороны через комната.

Ящик кладут на бок, а сторону с листом отдают стук рукой, как в барабан. Вихрь дымовых колец должен вылететь из отверстия и путешествовать по комнате. Процесс может быть повторять сколь угодно часто, пока остается немного дыма. После пока вся комната становится задымленной, и видимость ухудшается.Первое мало кольца обычно самые драматичные. Быстро движущееся кольцо может быть изготовлено сразу после более медленного, чтобы он догнал и обогнал медленнее. Можно построить два таких генератора вихрей и кольца проецируются друг на друга, чтобы изучить их взаимодействие. Должен быть заостренный выяснилось, что вихри существуют даже в отсутствие дыма, который единственная цель — сделать их видимыми. Это можно проиллюстрировать дует свечу через комнату.

Конструкция такого вихревого генератора делает дом идеальным. проект для людей всех возрастов. Зрителей можно посоветовать попробовать сделать один и исследовать эффект от использования отверстий разного размера и формы. Будет ли квадратное отверстие квадратным кольцом? Что происходит, когда кольцо прогнозируемый вверх?

ОБСУЖДЕНИЕ
Механизм образования вихревого кольца легко подобрать. понять.Когда воздух выходит из отверстия, он задерживается трением об край отверстие, заставляющее воздух в центре двигаться вперед быстрее, чем воздух на краю. Если вы представите, что едете вместе с вихрем, воздух в центр движется вперед, а воздух на краю движется назад. Это оставляет область пониженного давления перед кольцом на край и за кольцом в центре. Дополнительный воздух в центре передней части кривые наружу, а дополнительный воздух на краю сзади изгибается внутрь, чтобы уравнять давление, образуя кольцо.

Вихревые кольца — довольно частое явление в природе. Течет жидкости образуют вихри, когда они текут слишком быстро по узкому каналу, насколько это возможно видел наблюдая за быстрой рекой. Вихри на концах крыльев стремительный самолеты представляют опасность для других самолетов, которые случайно пролетают через их. Большинство людей видели, как курильщики создают вихревые кольца, формируя в рот в форме круга и мягко выдыхая. Когда-то считалось (неправильно), что взаимодействие атомов и их спектральное излучение можно понять с точки зрения вихревого движения внутри атома.

ОПАСНОСТИ
Опасности в основном связаны с дымом. Не следует размещать воспламеняется материалы внутри коробки. Когда задувает свечу через комната, свечу следует держать вдали от легковоспламеняющихся веществ.

2,18

Колокольчик

Объект, помещенный в колпак, подключенный к вакуумному насосу, расширяется при эвакуирован и сжиматься, когда воздух возвращается.
МАТЕРИАЛЫ
  • колпак *
  • вакуумный насос
  • воздушный шар
  • газированный напиток, зефир или крем для бритья (по желанию)
* Поставляется в American Scientific Products, Каролина Биологические Компания снабжения, Центральная научная компания, Фишер Сайентифик, Фрей Научный, Nasco и Sargent-Welch Scientific
ПРОЦЕДУРА
Воздушный шар наполняется воздухом примерно на половину внутреннего диаметра. колокола и завязал.Он находится внутри колпака. В аудитория просят предсказать, что произойдет, когда воздух будет удален из колокол банка. Включают вакуумный насос, откачивающий колпак. Воздушный шар расширяется. Затем аудитории предлагается предсказать, что произойдет, когда воздух повторно принят к колпаку. Насос выключается, и воздух поступает, что вызывает в воздушный шар, чтобы вернуть его почти к исходному размеру. Процедура может быть повторяется с другими предметами, такими как газированный напиток, зефир или бритье крем [1].Когда это делается с жидкостью, следует использовать ловушку серной кислоты. чтобы предотвратить загрязнение масла насоса.
ОБСУЖДЕНИЕ
Размер воздушного шара определяется равновесием между давление внутреннего газа и комбинированные эффекты давления внешний газ и эластичность материала баллона [2]. Воздушный шар расширяется когда внешнее давление уменьшается и сжимается, когда внешнее давление давление увеличена.Воздушный шар диаметром шесть дюймов на уровне моря имеет общая внешняя сила превышает три тонны! Следовательно, даже частичный вакуум существенно изменит свои размеры. Воздушный шар может не вернуться к вполне свой первоначальный размер, потому что материал воздушного шара может стать постоянно деформируется при расширении. По этой причине воздушный шар Полегче надуть второй раз, чем первый. Воздушный шар тоже легче надуть, если он сначала растягивается.

Зефир можно рассматривать как большое количество очень маленьких надувные шары. Большинство людей знает, что зефир в основном состоит из воздуха. Если зефир осталось в откачанном колпаке на несколько минут воздух будет в основном просачиваться вне захваченных карманов, и он будет сжат примерно до половины своего оригинал размер, когда воздух поступает достаточно быстро, чтобы карманы не пополнение. В газированных напитках диоксид углерода выходит из раствора. так как пониженного давления внешнего газа.Жидкость начнет кипятить если насос остается включенным слишком долго.

ОПАСНОСТИ
Основная опасность этой демонстрации — взрыв колпака. который находится под атмосферным давлением во время вакуумирования. Использовать только колокол банки, предназначенные для опорожнения, и будьте осторожны, чтобы не уронить колпак или ударьте по нему твердыми предметами.
ССЫЛКИ
1. Дж. Д. Уилсон, College Physics , Allyn and Bacon: Needham Высоты, MA (1989).

2. Дж. Уокер, Scientific American 261 , 136 (декабрь 1989 г.).


2,19

Жидкий азот

Объекты, помещенные в жидкий азот, меняют свои физические свойства так как пониженной температуры.
МАТЕРИАЛЫ
  • Дьюар жидкого азота *
  • пластиковый шланг или резиновый шарик
  • банан
  • щипцы
  • перчатки
  • молоток
  • стеклянная посуда
  • воздушный шар
* Жидкий азот можно приобрести в больничных магазинах.An обычный термос служит адекватным сосудом Дьюара, если он сделан из металл или стекло Pyrex (обычное стекло может лопнуть).
ПРОЦЕДУРА
На стол наливают немного жидкого азота, и публика Просили отметить, что стол не промок. Вещество, которое выглядит подобно вода является основным компонентом (80%) воздуха, которым мы дышим, но пониженная температура -196 ° C (-321 ° F, 77K). Это наверное в самое холодное вещество, которое когда-либо видели большинство людей.Можно объяснить, что это кипит без явного нагрева, потому что даже обычные материалы, такие как стол есть довольно много тепла, и не нужно много тепла, если кипятить азот.

One объясняет, что большинство материалов меняют свои физические свойства когда они простужаются. Большинство людей заметили, насколько твердые сиденья их машины холодным утром. Пластиковый шланг, погруженный в сосуд Дьюара. жидкость азот до прекращения кипения, а затем разбили молотком.Пока азот закипает, над сосудом Дьюара появляется небольшое облачко. В аудитория можно спросить, что это такое. Многие скажут «пар» или «дым», но о курс пар невидим, а дым — продукт горения. Облако фактически состоит из мелких частиц жидкой воды, конденсированной из воздуха над Дьюара за счет охлаждающего эффекта кипения азота.

Можно взять резиновый мяч, несколько раз отскочить им от пола и тогда бросьте его в жидкий азот.Когда кипение прекращается, мяч удалено щипцами, а затем в термоизолирующих перчатках бросить его на пол, заставив его разбиться. Как вариант, банан оставляют в жидкий азот в течение нескольких минут, а затем использовался для вождения большого гвоздь в блок пенополистирола. Банан имеет свойство ломаться и уходит довольно беспорядок, когда он тает.

Воздушный шар надувается примерно до шести дюймов в диаметре, привязан и помещают в стеклянную посуду.Баллон заливается жидким азотом вызывая он сморщится до незначительных размеров. Затем его снимают с блюда с щипцы или перчатки и удерживайте, пока он медленно надувается. Если смотреть внимательно, внутри воздушного шара можно увидеть жидкий воздух. Аудитория можно попросить объяснить, что происходит.

ОБСУЖДЕНИЕ
Температура жидкого азота достаточно низкая, чтобы материалы обладают существенно разными, а иногда и удивительными свойствами.Мало эластичный материалы сохраняют свою эластичность при этой температуре. Электрические сопротивление намного ниже, и некоторые материалы фактически становятся сверхпроводниками и терять все электрическое сопротивление.

Когда воздух в воздушном шаре охлаждается, его давление уменьшается, и давление внешней атмосферы давит его. Немного азота а также большая часть кислорода в баллоне может перейти в жидкое состояние.

ОПАСНОСТИ
Жидкий азот потенциально очень опасен, так как может вызвать мгновенное обморожение.Никогда не допускайте контакта с голой кожей. Объекты опускают в жидкий азот и снимают щипцами. Быть осторожный чтобы он не попал в глаза. Перчатки и защитные очки рекомендуемые. Когда предметы разбиваются молотком или метанием их об пол или стену полученные снаряды могут легко поранить кто-то, если не приняты соответствующие меры предосторожности

2,20

Симулятор кинетической теории

Набор небольших шарикоподшипников в корпусе, одна сторона которого является подключенные к громкоговорителю, демонстрируют движение, аналогичное движению молекул в газ.
МАТЕРИАЛЫ
  • симулятор кинетической теории *
* Коммерческие версии, иногда называемые «молекулярным движением» демонстранты » доступны от Central Scientific Company, Carolina Biological Поставка Company, Frey Scientific, PASCO Scientific и Sargent-Welch Научный.
ПРОЦЕДУРА
Дюжина или более маленьких пластиковых или стальных шариков помещаются в прозрачный контейнер, одна граница которого заставлена ​​быстро качаться назад и вперед вибратором или громкоговорителем, подключенным к мощному аудио осциллятор на частоте в несколько герц.Большую версию можно сделать с настольный теннис шары в большом цилиндре из оргстекла с моторным поршнем. Для большой аудитории, аппарат размещен на диапроекторе, затемнен с участием дуговую лампу напротив экрана или просмотр с помощью телекамеры и монитор. Амплитуду или частоту движущих колебаний можно изменить на проиллюстрировать эффект изменения температуры газа. Крышка отдыхающий на шарах, но свободно двигаться вверх, иллюстрирует оказываемое давление от молекулы в движении, когда они бомбардируют стенки своего контейнера.Один должен Подчеркните, что эта демонстрация является всего лишь симуляцией, поскольку типичные молекулы в газе примерно в десять миллионов раз меньше шариков используемый в демонстрации.
ОБСУЖДЕНИЕ
Подобные симуляции предоставляют полезные средства для визуализации движение молекул [1]. Идеальный газ подчиняется закону идеального газа

пВ = NkT

где p — давление, V — объем, N — количество молекулы в объеме k — постоянная Больцмана (1.38 x 10 -23 Дж / К) и T — температура (в Кельвинах). Таким образом, с заданным количеством молекулы (N) в постоянном объеме (V) давление увеличивается пропорционально в температура. Если объем (V) может изменяться, но давление (п) поддерживается постоянным за счет подвижной крышки заданного веса, объем (V) увеличивается пропорционально температуре. Это известно как Чарльза Закон Жака Александра Сезара Шарля (1746-1823).

Реальные газы обычно находятся в тепловом равновесии со стенками их контейнера, и, таким образом, молекулы в среднем будут получать как много энергия, поскольку они теряют при столкновении со стенами и друг с другом. В этом случае кинетическая энергия шаров значительно больше. чем их тепловая энергия при комнатной температуре (около kT) и, следовательно, шары теряют энергию каждый раз, когда сталкиваются. Следовательно, необходимо предоставить энергия вход, чтобы шары двигались.Эффективная «температура» смоделированный газ можно рассчитать из

кТ = mv 2

где m — масса шаров, а v — их скорость. Если мячи 0,01 кг и их скорость 10 м / с, температура около 10 23 градусов!

Эта демонстрация также показывает, как очевидная случайность может возникать из простых уравнений движения, если есть много частиц взаимодействуя.Даже без гравитации и с идеально упругими столкновениями (без энергии потеря), движение очень сложное и практически непредсказуемое после многих столкновения. Если бы один из шаров был окрашен в красный цвет и его движение последовал через множество столкновений он совершит случайное блуждание (броуновское движение).

ОПАСНОСТИ
Будьте осторожны, чтобы не сжечь динамик слишком большим входом вольтаж. Горстка шарикоподшипников, небрежно брошенная на пол, может коварная ходьба!
ССЫЛКА
1.Б.З. Шахашири, Химические демонстрации , Университет из Wisconsin Press: Мэдисон, Висконсин, Том 2 (1985).

2,21

Пожарный шланг нестабильности

Резиновый шланг, подключенный к источнику сжатого воздуха, свисает с потолок и крутится сложным образом.
МАТЕРИАЛЫ
  • резиновый шланг
  • источник сжатого воздуха
ПРОЦЕДУРА
Резиновый шланг подвешен к потолку одним открытым концом и выше уровня головы.Другой конец подключен к источнику сжатого воздух. Зрителей спрашивают, что бы произошло, если бы они включили садовый шланг на полную мощность, не цепляясь за его конец. Большинство людей ответить, дико размахивая руками. Отметьте, что этот эффект называется «нестабильность пожарного шланга», и что это может быть очень опасно с большой шланг, используемый пожарными. Затем включите сжатый воздух, чтобы иллюстрировать что с газами происходит то же самое, что и с жидкостями.Укажите, что это тот же самый сжатый воздух, которым можно было бы надуть автомобиль покрышка или велосипедная покрышка. Попробуйте шланги разного диаметра и с разные насадки, чтобы добиться наилучшего эффекта при длине и давлении. А аналогичный явление наблюдается при выпуске надутого воздухом баллона.
ОБСУЖДЕНИЕ
Шланг с выходящей жидкостью одним концом неустойчив, так же как карандаш уравновешенный по своей точке нестабилен. Если бы шланг был идеально прямым, сила реакции истощенной жидкости будет вдоль оси в шланг и просто сжимал его без каких-либо боковых движений.С жесткий шланг или труба, вот что произойдет. Это состояние является одним из неустойчивое равновесие, как карандаш, балансирующий вертикально на острие имеет всю силу вдоль своей оси. Однако, если шланг слегка изгибается, есть горизонтальная составляющая силы, которая заставляет его изгибаться Больше, и изгиб продолжает расти. Таким же образом, если карандаш не в яблочко вертикальный, есть компонент силы, которая заставляет его продолжать уходя от вертикали.

На самом деле нестабильность объясняет только начальное движение. После этого существует сложное взаимодействие силы реакции, гравитации и эластичный силы в шланге, которые приводят к хаотическому движению. В принципе, это было бы можно рассчитать результирующее движение, и с помощью компьютера его будет даже практичным, но предсказание будет точным только для короткая раз из-за крайней чувствительности к начальным условиям и к точному характеру сил.Как и все хаотические процессы, этот демонстрация иллюстрирует явно случайное поведение детерминированных систем.

ОПАСНОСТИ
Подвешивание шланга над уровнем головы предотвращает его попадание в лицо. Убедитесь, что он надежно подвешен. Увеличенные версии этого демонстрация использование газов и жидкостей под высоким давлением и больших шлангов впечатляет но влекут за собой значительные очевидные опасности.

2,22

Кран для капель

Капающий кран демонстрирует периодическое и хаотическое поведение, а период удваивающийся путь к хаосу.
МАТЕРИАЛЫ
  • водопроводный кран или клапан с точным управлением
  • источник воды
  • противень для выпечки из алюминия
  • микрофон, усилитель и динамик (дополнительно)
  • осциллограф (дополнительно)
ПРОЦЕДУРА
Поставьте алюминиевую форму для пирога нижней стороной вверх примерно на полметра ниже вода смеситель, скорость капания которого можно регулировать. Форма для пирога делает капает вода слышна.Микрофон, помещенный под форму для пирога и подключенный к в усилитель и динамик издают впечатляющий звук, легко слышимый большой аудитория. Выход микрофона также можно подключить к осциллограф настроена на медленную частоту развертки (около 1 Гц) и настроена на запуск по ввод звук. Телекамера, направленная на место падения капель. выпущенный улучшает видимость.

При низкой скорости стекания (скажем, раз в секунду) капает идеально. периодический.По мере того, как скорость капания медленно увеличивается, наступает момент где происходит бифуркация, и капание внезапно меняется на период дважды пока. Бифуркация очевидна в звуке, который должен измениться из «капать — капать — капать» на «капать капать — капать капать — капать капать». Осторожный корректировка и требуется терпение, чтобы обнаружить бифуркацию. Немного больше скорости стекания, возникает вторая бифуркация (в четыре раза больше периода), и поэтому четвертое, но эти более длительные периоды трудно наблюдать.В конце эта бесконечная последовательность удвоения периода, капает хаотично, и имеет нет обнаруживаемого образца. Его должно быть легко услышать и наблюдать на осциллограф хаотичное состояние.

ОБСУЖДЕНИЕ
Капающий кран — один из самых простых и распространенных примеров путь к хаосу удвоения периода. Это было широко учился [1-5], и об этом написана целая научная книга [6]. это простота делает его идеальным для домашних экспериментов.

Механизм следующий. Это максимальный размер капли. вес должен быть меньше, чем удерживающее его поверхностное натяжение все вместе и держит его прикрепленным к краю смесителя. Когда он достигнет этого критический По размеру капля разделяется на две части, одна из которых падает, а Другие остался позади. Оставшаяся капля затем начинает расти, и процесс повторяется. Если капля наполняется водой с постоянной скоростью, время требуется для достижения этого критического размера постоянно, и капли выпущенный периодически.Однако оставленная позади капля будет колебаться вверх и вниз. несколько раз, прежде чем остановиться, и инерция этой движущейся массы будет либо поощрять, либо препятствовать следующему выпуску капли в зависимости от фаза колебаний, когда капля достигает критического размера. А капля, выпущенная раньше, оставит больше воды для образования в следующая капля и наоборот. Разумно, что может быть состояние в котором чередующиеся капли находятся в фазе и противофазе с колебанием, соответственно, и это ситуация, которая приводит к удвоению периода.Хаос возникает, когда скорость капания выше, чем колебания частота так что фаза колебаний меняется каждый раз, когда капля выпущенный. Колебание можно легко увидеть, если внимательно присмотреться к крану.

Капли воды — полезная аналогия ядерным процессам. Eсть максимум размер ядра определяется конкуренцией электростатических отталкивание между протонами и ближней ядерной силой притяжения, которая ведет себя что-то вроде поверхностного натяжения.Когда ядро ​​превышает определенный критический размер, он расщепляется в процессе деления с большим выделением энергии. Осколки деления имеют разный размер, вероятно, определяемый фаза колебаний в ядре, когда происходит деление.

ОПАСНОСТИ
Единственная опасность — пролитая вода и, возможно, повреждение водой микрофон.
ССЫЛКИ
1. П. Мартиен, С. К. Поуп, П. Л. Скотт, Р. С. Шоу, Phys.Lett 110Z , г. 399 (1985).

2. X. Wu, E. Tekle, Z. A. Schelly, Rev. Sci. Instr. 60 , г. 3779 (1989).

3. X. Wu и Z. A. Schelly, Physica D 40 , 433 (1989).

4. Х. Йепез, Н. Нуние, А. Л. Салас Брито, К. А. Варгас и Л. А. Висенте, Евро. J. Phys. 10 , 99 (1989).

5. Р. Ф. Калалан, Х. Лейдекер и Г. Д. Кахалан, Comp. в Phys. 4 , г. 368 (1990).

6. Р. Шоу, Капающий кран как модель хаотической системы , Ариэль Пресса: Санта-Крус, Калифорния (1984).


[Следующая глава] [Предыдущая Глава] [Введение] [Сервер Дом]

Дж. К. Спротт

PPT — Расчет энтальпии Презентация PowerPoint, бесплатная загрузка

  • Расчет энтальпии И уравнение теплоемкости

  • Энтальпия • Общая энергия в системе • Если изменение положительное, эндотермическая реакция.• Если изменение отрицательное, реакция экзотермическая. • Изменение энтальпии составляет ∆H

  • Расчет энтальпии • В некоторых случаях мы даем значения и просто складываем их. • Фактически, энергия, необходимая для разрыва связей, добавляется к энергии, высвобождаемой при образовании новых связей. • Это самые простые!

  • Удельная теплоемкость • Поскольку в воде происходит так много реакций, мы измеряем изменение температуры воды. • Требуется ли вам столько энергии для нагрева 100 г воды на 5K, сколько потребуется для нагрева 100 г воздуха на такое же количество? • Зачем?

  • Снова склеивание • Из-за различий в склеивании многие вещества могут поглощать больше энергии, чем другие.• Это называется удельной теплоемкостью. • Вода имеет более высокую теплоемкость, чем воздух, и поэтому для повышения ее температуры требуется больше энергии. • Удельная теплоемкость воды составляет 4,18 Джг-1

  • Уравнение • Уравнение удельной теплоемкости: • DH = -смDT • c — удельная теплоемкость (4,18 Дж / г-1 или 4,18 кДж / кг) для воды. • m — масса (в г или кг). • DT — изменение температуры (всегда в К).

  • Используйте DH = -cmDT DH = -4.18 x 0,1 x 10 DH = — 4,18 кДж (c — удельная теплоемкость воды, 4,18 кДж кг-1oC-1) m — масса воды в кг, 0,1 кг DT — изменение температуры в oC, 10o) Энтальпия горение вещества — это количество энергии, выделяемое, когда один моль вещества сгорает в избытке кислорода. ХИМИЧЕСКИЕ РАСЧЕТЫ Рабочий пример 1. Сжигается 0,19 г метанола, Ch4OH, и отдаваемая тепловая энергия повышает температуру 100 г воды с 22 ° C до 32 ° C. Рассчитайте энтальпию сгорания метанола.Энтальпия горения. Используйте пропорцию для расчета количества тепла, выделяемого при сгорании 1 моль, 32 г метанола. 0,19 г -4,18 кДж. Итак, 32 г 32 / 0,19 x –4,18 = -704 кДж. Энтальпия сгорания метанола составляет –704 кДж / моль.

  • Расчет с использованием энтальпии сгорания Расчеты для вас. 1. 0,25 г этанола, C2H5OH, были сожжены, и выделенное тепло подняло температуру 500 см3 воды с 20,1 ° C до 23,4 ° C. Используйте DH = -cmDT DH = -4.18 x 0,5 x 3,3 = — 6,897 кДж Используйте пропорцию для расчета изменения энтальпии при сгорании 1 моль, 46 г, этанола. 0,25 г -6,897 кДж. Итак, 46 г 46 / 0,25 x -6,897 = -1269 кДж моль-1. Было сожжено 2,0,1 моль метана, и выделенная энергия подняла температуру 200 см3 воды с 18 ° C до 28,6 ° C. Рассчитайте энтальпию сгорания метана. Используйте DH = -cmDT DH = -4,18 x 0,2 x 10,6 = — 8,86 кДж. Используйте пропорцию для расчета изменения энтальпии при сгорании 1 моля метана.0,1 моль  -8,86 кДж Так 1 моль  1 / 0..1 x -8,86 = -88,6 кДж моль-1.

  • Переставьте DH = -cmDT, чтобы получить DT = DH -cm -11,1 -4,18 x 0,2 DT = = 13,3 oC Расчет с использованием энтальпии сгорания Рабочий пример 2. 0,22 г пропана использовали для нагрева 200 см3 воды при 20oC. Используйте энтальпию сгорания пропана из справочника для расчета конечной температуры воды. Сжигание 1 моль, 44 г, пропана DH = -2220 кДж. Пропорция сгорания 0.22 г пропана DH = 0,22 / 44 x –2220 = — 11,1 кДж Конечная температура воды = 20 + 13,3 = 33,3 ° C

  • Переставьте DH = -cmDT, чтобы получить DT = DH -cm -2,27 -4,18 x 0,1 DT = = 5,4 oC Расчет с использованием энтальпии сгорания Расчеты для вас. 1. Сжигали 0,1 г метанола, Ch4OH, и выделяемое тепло использовали для повышения температуры 100 см3 воды до 21 ° C. Используйте энтальпию сгорания метанола из буклета данных, чтобы рассчитать конечную температуру воды.Сжигание 1 моль (32 г) метанола DH = -727 кДж Пропорциональное сжигание 0,1 г метанола DH = 0..1 / 32 x -727 = — 2,27 кДж Конечная температура воды = 21 + 5,4 = 26,4 ° C

  • Переставьте DH = -cmDT, чтобы получить DT = DH -cm -11,14 -4,18 x 0,25 DT = = 10,66oC Расчет с использованием расчетов энтальпии сгорания для вас. 2. Было сожжено 0,2 г метана, Ch5, и выделенное тепло было использовано для повышения температуры 250 см3 воды. Используйте энтальпию сгорания метана в буклете данных для расчета повышения температуры воды.Сжигание 1 моль, 16 г метана DH = -891 кДж. Пропорционально сжигается 0,2 г метана. DH = 0..2 / 16 x –891 = — 11,14 кДж

  • Используйте DH = -cmDT DH = -4,18 x 0,1 x -3,3 DH = 1,38 кДж (c — удельная теплоемкость воды, 4,18 кДж. кг-1oC-1) m — масса воды в кг, 0,1 кг DT — изменение температуры в oC, -3,3oC) Энтальпия растворения вещества — это изменение энергии при растворении одного моля вещества в воде. ХИМИЧЕСКИЕ РАСЧЕТЫ Рабочий пример 1.5 г хлорида аммония, Nh5Cl, полностью растворяются в 100 см3 воды. Температура воды падает с 21 ° C до 17,7 ° C. Энтальпия раствора. Используйте пропорцию, чтобы найти изменение энтальпии для 1 моля растворенного хлорида аммония, 53,5 г. 5 г 1,38 кДж, поэтому 53,5 г 53,5 / 5 x 1,38 = 17,77 кДж моль-1. Примечание. — При понижении температуры реакция становится эндотермической (с выделением тепла) — на это указывает положительное значение изменения энтальпии.

  • Расчеты для вас. • 8 г нитрата аммония Nh5NO3 растворяют в 200 см3 воды. • Температура воды понижается с 20 ° C до 17,1 ° C. Используйте DH = -cmDT DH = -4,18 x 0,2 x -2,9 DH = 2,42 кДж. Используйте пропорцию, чтобы найти изменение энтальпии для 1 моля, 80 г, растворенного нитрата аммония. 8 г 2,42 кДж Таким образом, 80 г 80/8 x 2,42 = 24,2 кДж моль-1. 2. Когда 0,1 моль соединения растворяется в 100 см3 воды, температура воды повышается с 19 ° C до 22 ° C.4oC. Рассчитайте энтальпию растворения соединения. Используйте DH = -cmDT DH = -4,18 x 0,1 x 3,4 DH = -1,42 кДж. Используйте пропорцию, чтобы найти изменение энтальпии для 1 моля соединения. 0,1 моль  — 1,42 кДж. Итак, 1 моль  1 / 0,1 x -1,42 = -14,2 кДж моль-1.

  • Изменение порядка DH = -cmDT Дает DT = DH -cm 3,35 -4,18 x 0,15 DT = = -5,34 oC • Вы можете попробовать выполнить расчеты. • Энтальпия раствора хлорида калия KCl +16.75 кДж моль-1. • Как изменится температура, если 14,9 г хлорида калия • растворить в 150 см3 воды? Используйте пропорцию, чтобы найти изменение энтальпии для растворения 14,9 г хлорида калия. 74,6 г (1 моль)  16,75 кДж Итак 14,9 г 14,9 / 74,6 x 16,75 = 3,35 кДж

  • Используйте DH = -cmDT DH = -4,18 x 0,2 x 6,2 DH = — 5,18 кДж (c — теплоемкость емкость воды, 4,18 кДж кг-1oC-1) м — масса смешанного раствора в кг, 0.2 кг DT — изменение температуры в ° C, 6,2 ° C) Энтальпия нейтрализации вещества — это количество энергии, выделяемой при образовании одного моля воды в реакции нейтрализации. ХИМИЧЕСКИЕ РАСЧЕТЫ Рабочий пример 1. 100 см3 1 моль / л соляной кислоты HCl смешивали с 100 см3 1 моль -1 гидроксида натрия, NaOH, и температура повышалась на 6,2 ° C. Энтальпия нейтрализации. Уравнение реакции: HCl + NaOH  NaCl + h3O. Число молей используемой кислоты = Число молей щелочи = C x V (в литрах) = 1 x 0.1 = 0,1 Таким образом, количество образовавшихся молей воды = 0,1 Используйте пропорцию, чтобы найти количество тепла, отдаваемого при образовании 1 моля воды. 0,1 моль  -5,18 кДж. Итак, 1 моль  1 / 0,1 x –5,18 = -51,8 кДж моль-1.

  • Расчет с использованием энтальпии нейтрализации. • Расчеты для вас. • 400 см3 0,5 моль л-1 соляной кислоты. HCl, прореагировала с 400 см3 • 0,5 моль / л гидроксида калия, и температура повысилась на 3,2 ° C. • Рассчитайте энтальпию нейтрализации.Используйте DH = -cmDT DH = -4,18 x 0,8 x 3,2 DH = — 10,70 кДж Уравнение реакции: HCl + KOH  KCl + h3O. Число используемых молей кислоты = Число молей щелочи = C x V (в литрах ) = 0,5 x 0,4 = 0,2 Таким образом, количество образовавшихся молей воды = 0,2. Используйте пропорцию, чтобы найти количество тепла, выделяемого при образовании 1 моля воды. 0,2 моль  -10,70 кДж. Итак, 1 моль  1 / 0,2 x -10,70 = -53,5 кДж моль-1.

  • Расчет с использованием энтальпии нейтрализации. Расчеты для вас. 2. 250 см3 0,5 моль л-1 серной кислоты. h3SO4, прореагировала с 500 см3 0,5 моль / л гидроксида калия, и температура повысилась на 2,1 ° C. Рассчитайте энтальпию нейтрализации. Используйте DH = -cmDT DH = -4,18 x 0,75 x 2,1 DH = — 6,58 кДж Уравнение реакции: h3SO4 + 2NaOH  Na2SO4 + 2h3O. 1 моль кислоты реагирует с 2 молями щелочи с образованием 1 моля воды. Количество используемых молей кислоты = 0,5 x 0,25 = 0.125 Количество используемых молей щелочи = 0,5 x 0,5 = 0,25 Таким образом, количество образовавшихся молей воды = 0,125 Используйте пропорцию, чтобы найти количество тепла, выделяемого при образовании 1 моля воды.

  • Leave a Reply

    Добавить комментарий

    Ваш адрес email не будет опубликован. Обязательные поля помечены *